Sie sind auf Seite 1von 36

1AC

1AC Plan
The United States federal government should cooperate with the People’s
Republic of China and the Russian Federation to negotiate an international
agreement that bans debris-producing destruction or damage of orbiting space
systems.
1AC Arms Race
The rising trend of kinetic ASAT arms racing is catastrophic—inadvertent
nuclear escalation is extremely likely in a case where ASAT capabilities strike
early warning communications satellites.
The threat of pre-emptive strikes encourages escalation to nuclear use.

Johnson-Freese 17—Joan Johnson-Freese, Professor and the Charles F. Bolden, Jr. Chair of
Science, Space & Technology at the Naval War College, Ph.D., Kent State University, 1981,
Political Science/International Relations (“Protecting Space Assets,” Chapter 1 of Space Warfare
in the 21st Century: Arming the Heavens, Routledge)

Space warfare runs two untenable risks: the creation of destructive debris and escalation to
terrestrial, even nuclear, warfare. Kinetic warfare in space creates debris traveling at a speed of more than
17,000 miles per hour, which then in itself becomes a destructive weapon if it hits another object—even
potentially triggering the so-called Kessler Syndrome,86 exaggerated for dramatic effect in the movie Gravity. Ironically, both China
and the United States learned the negative lessons of debris creation the hard way. In 1985, the United States tested a miniature
homing vehicle (MHV) ASAT launched from an F-15 aircraft. The MHV intercepted and destroyed a defunct US satellite at an altitude
of approximately 250 miles. It took almost 17 years for the debris resulting from that test to be fully eliminated by conflagration re-
entering the Earth’s atmosphere or being consumed by frictional forces, though no fragment had any adverse consequences to
another satellite—in particular, no collisions. China irresponsibly tested a direct-ascent ASAT in 2007, destroying one if its defunct
satellites. That test was at an altitude almost twice that of the 1985 US test. The debris created by the impact added 25 percent to
the debris total in low Earth orbit87 and will dissipate through the low Earth orbit, heavily populated with satellites, for decades,
perhaps centuries, to come. Perhaps most ironically, because of superior US debris-tracking capabilities, the United States—even
though not required to do so—has on more than one occasion warned China that it needed to maneuver one of its satellites to avoid
a collision with debris China itself had likely created.88 In 2013, a piece of Chinese space junk from the 2007 ASAT test collided with
a Russian laser ranging nanosatellite called BLITS, creating still more debris.89 The broader point is that all
nations have a
compelling common interest in avoiding the massive increase in space debris that would be
created by a substantial ASAT conflict.
Gen. Hyten has said that not creating debris is “the one limiting factor” to space war. “Whatever you do,” he warns, “don’t create
debris.”90 While that might appear an obvious “limiting factor,” preparing to fight its way through a debris cloud had been a
Pentagon consideration in the past. Now, however, sustaining the space environment has been incorporated
into Pentagon space goals.

Beyond debris creation, MacDonald points out that as


China becomes more militarily capable in space and there
is more symmetry between the countries, other risks are created – specifically, escalation.

That is, theUnited States could threaten to attack not just Chinese space assets, but also ground-based assets,
including ASAT command-and-control centers and other military capabilities. But such actions, which would
involve attacking Chinese soil and likely causing substantial direct casualties, would politically weigh much heavier than the U.S. loss
of space hardware, and thus might climb the escalatory ladder to a more damaging war that both sides would
probably want to avoid.91

MacDonald isn’t alone in concerns about escalation. Secure World Foundation analyst Victoria Samson has also voiced apprehension
regarding US rhetoric that does not distinguish between actions against unclassified and classified US
satellites, stating that “things can escalate pretty quickly should we come into a time of
hostility.”92

Theresa Hitchens explained the most frightening, but not implausible, risk of space war escalation in a
2012 Time magazine interview.
Say you have a crisis between two nuclear-armed, space-faring countries, Nation A and Nation B, which have a long-standing border
dispute. Nation A, with its satellite capability, sees that Nation B is mobilizing troops and opening up military depots
in a region where things are very tense already, on the tipping point. Nation A thinks: “That’s it, they’re going to
attack.” So it might decide to pre-emptively strike the communications satellite used by Nation B to
slow down its ability to move toward the border and give itself time to fortify. Say this happens and Nation B has no use of satellites
for 12 hours, the time it takes it to get another satellite into position. What
does Nation B do? It’s blind, it’s deaf, it’s
thinking all this time that it’s about to be overwhelmed by an invasion or even nuked. This is possibly a
real crisis escalation situation; something similar has been played out in U.S. Air Force war games, a scenario-planning
exercise practiced by the U.S. military. The first game involving anti-satellite weapons stopped in five
minutes because it went nuclear – bam. Nation B nuked Nation A. This is not a far-out, “The sky’s falling in!”
concern, it is something that has been played out over and over again in the gaming of these things, and I have real fears about it.93

While escalation
to a nuclear exchange may seem unthinkable, in war games conducted by the military,
nuclear weapons are treated as just another warfighting weapon.

Continued space arms race ends strategic stability ---it makes the risk of
accidental nuclear launch unacceptable---ONLY clarity and certainty over early
warning satellites maintains deterrence
Arbatov 19 — Alexey Arbatov (head of the Center for International Security at the Primakov National Research Institute of
World Economy and International Relations, PhD, History, Moscow State Institute of International Relations, MA, Moscow State
Institute of International Relations, former scholar in residence and chair of the Carnegie Moscow Center’s Nonproliferation
Program, member of the research council of the Russian Ministry of Foreign Affairs, the governing board of Stockholm International
Peace Research Institute, the Center for Nonproliferation Studies at the Monterey Institute, and the Russian Council for Foreign and
Defense Policy), 6-28-2019, “Arms control in outer space: The Russian angle, and a possible way forward,” Bulletin of the Atomic
Scientists, Volume 75, pages 157-158, 2019, https://doi.org/10.1080/00963402.2019.1628475, [accessed: 8/1/19]
Prospects for agreements

Traditionally, practical (in contrast to romantic) disarmament stemmed from emergence of the overlapping strategic interests of the parties. Proceeding from common interest, initial US-Russian space arms
limitation may capitalize on strategic stability the concept of , which for 20 years served as the basis for START treaties.44 The concept of strategic stability was

formalized in the US-Soviet Declaration of 1990 ,45 which set out a goal: a state of strategic relationship under which the incentives for a first nuclear strike by either the United
States or Russia were removed.

Space arms control efforts may be related to strategic stability, first of all, if theyaim at enhancing survivability of early-warning
the the

satellites that provide info rmation about foreign ballistic missile launches 90 seconds after their start . Information about the origin,

Russia sustains it as the


scale, and initial direction of a missile attack is indispensable for retaliation in the form of launch-onwarning. This type of response is one of the operational options of both nuclear superpowers, but

principal concept of its nuclear deterrence strategy.46

A missile attack should also be confirmed by land based radars 10–15 minutes after its detection by satellites. After that, the leadership of the attacked nation would have several minutes to take political decision and technically authorize the launch of its own

Early warning
combat ready ICBMs and, for Russia, also for the firing of some missiles on submarines that are in their bases, rather than at sea. by satellites also allows alerted strategic aircraft to
immediately take off before they are destroyed at the airfields (leaving the possibility of calling them back in case of a false alarm).

With boost-glide hypersonic systems ICBMs might be launched on warning


the advent of , the on the basis of satellites’ detection of a

land-based radars unable to provide timely confirmation


hostile missile launch alone, beingASAT of an incoming hypersonic attack.47 Thus, an

threat to early-warning satellites would increase the risk of inadvertent or accidental nuclear tangibly

war caused by false alarm or interruption of communication with satellites


the firing missiles as a result of .

launch-on-warning
The creates a threat of nuclear war that might erupt due to a
concept is controversial and risky; it

technical malfunction, unauthorized actions, or a political miscalculation. even if both 48 But

countries abandon this operational concept, early warning


were to space systems would remain crucial for nuclear deterrence and strategic stability. They

provide timely warning to national leaders so they can take cover or delegate (underground or in the air)
authority for implementing retaliation to survivable command posts; make decisions that will increase the survivability of nuclear forces (such as dispersing ground-mobile ICBMs and putting aircraft
in the air); and have full understanding on the origin and scale of attack.

leaders will have time to make a thought-through decision


Possessing this information, on the scale and targets of a delayed second strike or any other response

This capability provided


steps. by early warning satellites reduces the incentives for a nuclear
– in large part –

first strike and enhances mutual nuclear deterrence and strategic stability.

Early-warning satellites also play a role in ballistic missile defenses. For many years, the assessment of anti-missile defenses as destabilizing or stabilizing has been a subject of great controversy between, on the one hand, the United States and its allies and, on the
other, Russia and China. Still, even if ballistic missile defense systems are considered destabilizing, there are numerous ways to defeat them besides putting early-warning satellites in jeopardy – which would be destabilizing no matter what.

Satellites of this class are located at geostationary or highly elliptical orbits.


There is good news in this context: At the moment, Russia

neither the
is reconstituting its constellation with the deployment of the Unified Space System. The United States meanwhile is replacing its old Defense Support Program satellites with the new Space-Based Infrared System. To date,

United States nor Russia has operationally deployed dedicated high-orbit anti-satellite weapons .

Those available are dual-purpose systems an element of (the US sea-based Aegis SM-3 and Russia’s land-based Nudol’ and forthcoming S-500 Prometey ballistic missile defense systems).

the dual-purpose nature of


On the one hand, ASAT systems makes their verifiable ban or
US and Russia’s main existing

limitation highly problematic. On the other hand, the existing systems are primarily for low-orbit interception ,
except for surveillance and space awareness satellites that are on high orbits and may be used to attack other satellites.

The aff’s ban on kinetic ASATs solves- focusing on hit-to-kill operations is the
first step in getting US, Russia, and China on the same page.
- High-value satellites for communication, navigation, and missile warning are higher up
- Arms-race of kinetic ASATs – testing creates debris jeopardizes satellites
- Ban on hit-to-kill is verifiable, prevents debris causing tests, and slows development

Lewis 14—Jeffrey Lewis, director of the East Asia Nonproliferation Program for the James
Martin Center for Nonproliferation Studies at the Middlebury Institute of International Studies
at Monterey (“They Shoot Satellites, Don’t They?” Foreign Policy, August 9th,
https://foreignpolicy.com/2014/08/09/they-shoot-satellites-dont-they/)

China’s
Hit-to-kill is really just an advanced defense technology. There is a lot of talk about China pursuing "asymmetric" technologies to threaten the United States. But this is a case where

interest is totally symmetric. The Chinese are interested in hit-to-kill for the same reasons as the
United States is. In fact, the Chinese are probably doubly interested because they don’t want to be left behind as the United States develops an important new defense technology that can be used
to intercept anything — airplanes, missiles, and satellites. China might have shot down a missile during its last test, but it is really building up a broad technological capability that can be used for any number of

What Beijing ultimately chooses to do with its hit-to-kill capability, once the country has it, depends on a
missions.

lot of factors, not all of which are under our control.

It’s not only the United States and China . Russia, as well as many of America’s allies like Japan and Israel, are
investing in hit-to-kill systems that can be used for missile defense or anti-satellite applications. This is the basic technology that Israel uses in Iron Dome to intercept short-
range rockets. (Iron Dome doesn’t have any capability against satellites, but Israel’s Arrow system does, as might longer-range versions derived from Iron Dome.) Even India is getting

into the act.

the proliferation of hit-to-kill technologies means that many,


Which brings us to the problem. Whatever one thinks of missile defenses,

many countries are going to have very fancy anti-satellite weapons in the not-so-distant future.

Satellites are far more vulnerable than ballistic missiles — and a lot more important. China will never come
under ballistic missile attack from the United States unless America is starting a nuclear war. That’s not very likely. On the other hand, in a conventional conflict over

Taiwan or some rock outcropping doomed by global warming, China might be tempted to start knocking down U.S. or
Japanese satellites. No country depends on satellites for military operations more than the
United States do. The problem is that many countries also depend on these satellites for a range of civilian
applications including communications and navigation. And all of these assets would be jeopardized by the debris from anti-
satellite test strikes.
For a long time, some people argued that the proliferation of hit-to-kill systems might be a problem — but only for satellites in low-Earth orbit. These are mostly (though not exclusively) the satellites that take the

The really high-value civilian and


sort of pictures you find in Google Earth. The International Space Station and Hubble Space Telescope are in this orbit too.

military satellites are used for communication, navigation, and missile warning. These are
higher up, with navigation satellites in medium-Earth orbit and satellites for missile warning and communications higher still in geosynchronous orbit. Which brings us to an ambiguous, but distressing
event.

In May 2013, China conducted a curious high-altitude scientific experiment, sending a sounding rocket into the
magnetosphere to release a cloud of barium powder. Although Beijing has a real research program to study the magnetosphere, this launch seemed different in a lot of important ways, from how it was conducted

to the secrecy surrounding it. The evidence is really quite ambiguous, but as my colleague Brian Weeden argues in a very long essay, the simplest explanation is that it was a test of a hit-
to-kill system that can reach geostationary orbit.

If China is developing an anti-satellite capability to hold at risk all U.S. satellites , this is a very
distressing development. After the mid-1980s, the United States gradually lost interest in developing
debris-creating anti-satellite weapons because it has the most to lose if space becomes so littered
with debris that it is unusable. China and Russia, on the other hand, might not be so delicate if push came to shove. A world filled
with hit-to-kill anti-satellite missiles should be very disturbing.
So what to do?

One option is to negotiate some sort of ban on hit-to-kill anti-satellite weapons. Washington and Moscow tried to do this in
the 1970s, but issues of definition and verification were pretty difficult. Remember the U.S. missile defense system modified to shoot down USA-193? How would the Russians know that software isn’t installed on
every U.S. Aegis destroyer?

Despite these problems, there are some verifiable partial measures, such as a ban on any hit-to-kill
test that creates orbital debris. That won’t stop the development of hit-to-kill capabilities for missile defense — but it would mean that
China’s newest SC-19 and any follow-on systems would remain untested against a real target.
That’s not much, but it’s something — and a place to start.

In 2008, then President-elect Barack Obama actually came out in support of such an agreement. That’s right, he filled out a little survey for the Arms Control Association that included the sentence: "I will pursue
negotiations of an agreement that would ban testing anti-satellite weapons." (Full disclosure: I volunteered on the team that actually filled out the survey for him. I hope you aren’t scandalized that presidents
don’t fill out questionnaires by themselves or that people try to put words in the president’s mouth!)

The administration simply hasn’t followed up on the idea of negotiating a ban on anti-satellite testing, despite the importance of protecting U.S. assets in space. The administration has stated, in its National Space
Policy, that it "will consider proposals and concepts for arms control measures if they are equitable, effectively verifiable, and enhance the national security of the United States and its allies." This is an
improvement over the 2006 edition, as U.S. officials argue, but it is still a passive position of waiting for someone else to come up with an idea to solve the problem. And it sure as hell doesn’t do anything to
prevent China from developing ever better anti-satellite missile technology.

Still, there are already a number of proposals outlined by various experts who share my concern about the proliferation of hit-to-kill technologies and the threat to space assets. So, perhaps it is time that the
administration consider some of them. Bruce MacDonald wrote a report for the Council on Foreign Relations that contained the model for such a treaty. Geoffrey Forden outlined a slightly different version in
Arms Control Today, while the Stimson Center’s Michael Krepon has suggested that the United States, Russia, and China could agree to a moratorium on "further [anti-satellite] tests that generate long-lived,
indiscriminately lethal space debris" as part of the now-stalled effort to develop a code of conduct for space activities. I have even offered my own meager thoughts on such an agreement in exchange for a few
days per diem in Geneva.

The short version of all the proposals is simple : no blowing things up in space that leaves a
bunch of space junk. All of these proposals aim to prevent the creation of yet more debris that will
threaten the commercial, civil, and military uses of space , while also trying to slow the
development of dedicated anti-satellite systems. States will still develop hit-to-kill systems — like the United States, Russia, and China — and have a latent capability
to threaten space assets. But the least we can do is prevent a race to start testing these systems against live

targets in orbit.

There is value to limiting anti-


I think a fair-minded observer would conclude that such a treaty certainly meets the criteria outlined in the National Space Policy.

satellite testing; the creation of debris is easily verified; and it would not limit U.S. missile
defense programs. But perhaps most importantly, a ban on anti-satellite tests that create debris in space would be more in the interest of the United States than any other country on Earth.

Even a small nuclear exchange kills millions and results in global famine
Erin Hunt 18, Program Manager at Mines Action Canada, working in humanitarian
disarmament in various capacities since 2006 and doing public education on the Ottawa Treaty
banning landmines since 2003, 6/25/18, “Sustaining Destruction: Nuclear Weapons and the
Sustainable Development Goals,” https://impakter.com/sustaining-destruction-nuclear-
weapons-sustainable-development-goals/

A single nuclear weapon detonated in a major city could kill millions and continue to cause
harm to people and the environment for decades . The humanitarian community knows that no relief efforts will be possible in the face of a nuclear
detonation, no state, agency or international organization has the capacity to respond to the

devastation caused by a nuclear weapon. A limited nuclear exchange is predicted to result in a global famine
that would kill two billion people due to the soot deposited in the atmosphere . That soot would alter
temperatures and decrease food production around the world . Costs would skyrocket and the most

vulnerable people who are already malnourished or food insecure would no longer be able to
afford food. As food prices increase and famine sets in, people will be forced to migrate to survive thus increasing pressure on the few areas still able to produce food.
While the international community is taking steps to address climate change, there are significantly fewer plans to address the threat posed by nuclear weapons. This lack of action is problematic because

nuclear weapons also hamper other global efforts like the Sustainable Development Goals (SDGs). As a universal call to action to end poverty,
protect the planet and ensure that all people enjoy peace and prosperity, the SDGs set an ambitious and inclusive agenda for
development. All states have committed to working towards these targets by 2030. However, a commitment to the SDGs is completely incompatible with the use, development or possession of nuclear weapons
or the reliance on nuclear deterrence.

nuclear conflict would have disastrous consequences for global development . Global
As mentioned above, a limited

famine would make the achievement of the SDGs impossible. Clearly, the SDGs around reducing poverty and hunger, increasing health and human rights, and

building economic security for all would see dramatic declines in a situation where the majority of
the planet is struggling to feed themselves.

Chinese and Russian action is reactionary to US – absent diplomacy and arms


control, great power competition will consume space. Military modernization in
space drives arms racing and inevitable conflict—the aff’s narrow agreement
solves.
Lamrani 18—Omar Lamrani, Senior Military Analyst, Stratfor, master's degree from the
Diplomatic Academy of Vienna (“Space: The Final Frontier for War?” RealClearDefense,
September 25th,
https://www.realcleardefense.com/articles/2018/09/25/space_the_final_frontier_for_war_113
835.html)

The U.S. move to beef up its defenses in space is understandable given the increasing attention the frontier is
receiving from its closest competitors, China and Russia. Both countries continue to develop their anti-satellite capabilities,
including specialized maneuvering satellites that could be deployed to either conduct repairs on other satellites or interfere with them in a nefarious way. Nevertheless, in the

absence of parallel diplomatic efforts and a wider comprehensive strategy to establish international standards for
conduct in space, the U.S. move to strengthen its defenses in space could drive the Russians and the Chinese to
accelerate their own space militarization efforts. For instance, the United States could technically use
interceptors, which are essential to a space-based ballistic missile defense system, to destroy other satellites — a fact that is bound to
drive Beijing and Moscow to establish their own deterrents in space in the absence of
significant transparency or agreements.
The 1967 Outer Space Treaty is the world's foundational and governing document on space. The agreement bars members (including the United States, Russia and China) from placing nuclear weapons in space
and mandates only non-military activity on the moon and other celestial bodies. The pact does not, however, enact restrictions on the placement of conventional weapons in space, and subsequent efforts to
impose such limits, including the 2006 Space Preservation Treaty, have failed.

Such blanket treaties are unlikely to succeed in the foreseeable future because of their significant limitations, as well as concerns over the ability to verify
compliance. The United States has especially voiced concern in this regard, repeatedly voting down such initiatives in addition to noting that the treaties fail to take into account the ability of terrestrial anti-
transparency and
satellite weapons to damage space assets. Nevertheless, the dim prospect of any blanket ban on weapons in space should not limit the effort to engage in

confidence-building measures, which could provide a foundation for more narrow arms control
agreements that limit specific aspects of the increasing militarization of space or the scope of such activities over the
long term.

Whether or not U.S. defense in space is best served by the creation of a new service is an organizational question that will attract debate over the next few years both inside and outside the military. What the

any space
question does not address is the need to incorporate a simultaneous and concerted effort to establish international norms in space to prevent a destabilizing, militarized space race. Because

conflict would produce large amounts of space debris that would ruin the orbiting assets of all
countries — and devastate the world economy in the process — the only way to win a war in space is to not fight one. To that end, space should increasingly be
seen through a prism akin to that of nuclear deterrence in which the ultimate aim is to prevent a
space war from ever occurring. There is a vital need to build up a defensive capacity to deter an attack in space, but this capacity should be
tempered by outer space treaties and norms in much the same way as arms control agreements have played a vital role in restricting nuclear arsenals. The
future of the world and its neighborhood depends on it.
1AC Solvency
The aff solves – China and Russia say yes, and other countries get on board.
Arbatov 19 (Alexey, Center for International Security at the Institute of World Economy and
International Relations of the Russian Academy of Sciences, member of the advisory council to
the Russian foreign minister and an adviser on defense and security to the head of the Center
for Strategic Research, member of the governing board of the Nuclear Threat Initiative and of
the James Martin Center for Nonproliferation Studies at the Middlebury Institute of
International Studies at Monterey, former deputy chair of Defense Committee in Russian
parliament, former head of nuclear non-proliferation at Carnegie Moscow Center, “Arms control
in outer space: The Russian angle, and a possible way forward,” Bulletin of the Atomic Scientists,
Volume 75, Issur 4, 2019, Taylor & Francis)

By the logic of the past ABM, SALT, and START agreements, an obvious common interest of the United States and Russia in
space should be to limit as much as possible (if total prohibition is unfeasible) the dedicated anti-satellite systems that
threaten the early-warning satellites of each nation. The negotiations on these ASAT systems
should be narrow, at least initially, to avoid overburdening the diplomatic process. This approach would leave out some
systems and perceived threats that are difficult to technically define or that pose huge verification problems (i.e. lasers, particle-beams, electronic jamming, cyber systems). But this method of focusing

on the ASAT systems whose prohibition or limitation could be verified would facilitate the start
of the process, like limiting the number of missile launchers available to each country did 50
years ago at the SALT-I talks.

The ability to elaborate realistic and reliable verification means is crucial to the success of
practical arms control. No doubt, it would be preferable to prohibit deployment of high-orbit anti-satellite systems, regardless of their destruction mechanism or basing modes – but for
technical reasons, this would be very difficult to do in a verifiable way. The development of reliable, direct-ascent ASAT systems to

attack early warning satellites in high orbits, however, would require major programs and realistic
testing that would be impossible to conceal.

Instead of prohibiting deployment of such high-orbit systems, the parties could resolve this problem indirectly, agreeing as a first step to
prohibit testing of anti-satellite systems involving the actual destruction of target satellites (space
objects) – that is, the kind of tests conducted by the USSR in 1960s and the 1980s, by the United States in the 1980s and 2008, by China in 2007 and by India in 2019. This test ban would not affect

already tested ASAT systems, but would seriously hamper the development of high-orbit follow-on weapons. At

the same time, such a ban would limit the growing volume of space debris threatening the
satellites of all nations.

Verification of such an agreement could be based on national technical means, preferably in conjunction with
cooperative measures and well-defined transparency. For example, the existing format for notification
of all missile launches should be expanded to all experiments that have a destructive effect on
space objects. Elimination of obsolescent satellites that threaten to fall to Earth should be conducted under the supervision of the other party or parties, with sufficient information presented so as
to remove suspicion of tacit ASAT testing.

Rendezvous and proximity operations – in which one satellite is maneuvered close to another – may gradually enhance anti-satellite capabilities and
eventually come to threaten early-warning satellites. This threat may be addressed at a later stage of talks.
A partial solution might consist of an agreement that would strictly regulate the permitted speed and distance of such approaches
toward all space objects and require pre-notification of all such operations.
The first treaty could initially involve the United States, Russia, and, one might hope, China and India, but also provide for
the possibility of membership for any other country in the future. The treaty could be limited to a 10-year
period, with the option of extending. This period would be less than the time required for the
deployment of any technically feasible space-based ballistic missile defense system that would be a primary
concern and target of Russian (and Chinese) land-based ASAT programs. That is why banning anti-
satellite tests might be acceptable to them. Besides, as with any other such agreement, this first anti-satellite weapon treaty should contain the right of
withdrawal, should an extraordinary event jeopardize the “supreme interests” of either side. Russia (and China, if it joined such an agreement) could make a unilateral statement referring to the creation of space
ballistic missile defense as a possible cause for invoking the withdrawal right.49

FOOTNOTE 49. If other nations follow India’s 2019 ASAT test example, it might be politically difficult to preserve a bi- or trilateral ASAT test ban. However, strategically this should not be an obstacle: the
deterrence to ASAT development by third states may rely on other weapon systems and
strategies rather than development of analogous anti-satellite arms (all the more so that third states would hardly have similar
high-value space systems of their own as objects for retaliation).

this proposal appears to be relatively


The proposed treaty has obvious deficiencies, but its advantages clearly outweigh its shortcomings. Perhaps most important,

feasible as a practical first step in preventing the weaponization of outer space. It is


strategically attractive to the parties, it has clear technical properties that could reasonably be addressed, and
compliance with those regulations could be verified through national technical means and
cooperative measures already in use.

the alternative to the


One final argument in favor of the proposed treaty: the apparent absence of realistic alternatives for initiating measures that promote space non-weaponization. In reality,

would be no legal restrictions on the weaponization of outer space, which would


proposed treaty

gradually transform it into an arena for arms racing and , potentially, armed conflicts.

A limited ban is verifiable and discourages countries from developing new


weapons and using existing ASATs.
Koplow 18—David A. Koplow, Professor of Law, Georgetown University Law Center (“The
Fault Is Not in Our Stars: Avoiding an Arms Race in Outer Space,” Harvard International Law
Journal, Vol. 59, No. 2, Summer 2018,
https://scholarship.law.georgetown.edu/cgi/viewcontent.cgi?article=3101&context=facpub)

Turning now to the positive attributes of the proposal, this test-ban regime would pursue two principal
purposes, in addition to the general overall goal of seeking to reinvigorate the long-moribund
international dialog on space security and arms control. The first objective is to inhibit the
development of at least one type of ASAT technology . The concept here rests upon the “test-ban
theory of arms control”: countries will be reluctant to rely upon weapons that have not been
thoroughly tested, so the most effective mechanism for inhibiting the next spin in an arms race is to
deprive them of the opportunity to assure themselves that the new evolving prototypes will , in
fact, operate as intended. In this view, blocking future qualitative improvements in a weapon may be
even more important than capping the sheer numbers of those devices, and a test ban can be
the key governor against innovations.126 This theory is not iron-clad; states sometimes do invest in weapons that
have not survived the crucible of rigorous testing.127 But prudence strongly guides them in the opposite direction: a
cautious budget office will be reluctant to devote scarce resources to procurement of unproven
systems, and conservative military and civilian leaders will hesitate to rely upon arms of unproven effect. The history of
weapons development is littered with illustrations of conspicuous and expensive failures
resulting from decisions to forego customarily exhaustive testing , especially where new technology is
pushing the envelope of prior experience.128 In addition to helping retard the proliferation of ASAT capabilities, this test-ban
regime could, over time, reduce the confidence of countries that have already completed their
testing of debris-creating ASAT systems. That is, even a proven military strength can atrophy if it is not
regularly exercised,129 and space weapons would be no exception. The U.S. and Soviet legacy kinetic-
energy interceptor systems that were deemed operational in the 1980s, for example, largely
deteriorated through desuetude in the subsequent decades, and their more recent ASAT enterprises had to start
more or less from scratch.130

The logic of arms control in space adopted by the plan hamstrings the U.S.
pursuit of national domination – an international legal regime would prevent
imperial expansion into space in favor of binding commitment to equal access.
Tannenwald ‘4 Nina, Joukowsky Family Research Assistant Professor and Director of the
International Relations Program, Watson Institute for International Studies, Brown University.
Law Versus Power on the High Frontier: The Case for a Rule-Based Regime for Outer Space, 29
Yale J. Int'l L. (2004).

The dominant challenge to the future of outer space lies in the existence of two competing
visions of how activities in space should be organized, managed, and controlled. The first view
emphasizes the central role of law in preserving space for peaceful purposes and in promoting
international cooperation in the use and exploitation of space for the benefit of all. This view emphasizes the benefits of a
multilateral legal regime as the best way to balance the various interests in space, to manage the possible interference of activities,
and to ensure that no single power dominates and possibly jeopardizes access to space by others. Power is constrained by
law, and national interests are pursued in the context of a developed and articulated legal framework and an assumption of mutual
and reciprocal interests. This is the logic of the current legal regime for space (however weak and incomplete),
as reflected in a set of outer space, arms control, and commercial treaties and agreements
beginning in the 1960s.3 The second view is the logic of national dominance projected by the former U.S. Space
Command (SPACECOM).4 With the United States increasingly reliant on space for both commercial and military support activities,
SPACECOM argues that U.S. assets in space are vulnerable to attack and that, in order to protect them, the United States
needs to dominate space militarily . SPACECOM's Visionfor 2020 argues that the protection of space requires superior
U.S. space warfare capability and proclaims its members "stewards for military space." 5 It sets out two principal themes: (1)
dominating the space dimension of military operations to protect U.S. interests and investment; and (2) integrating space forces into
warfighting capabilities across the full spectrum of conflict. 6 As Air Force General Joseph W. Ashy, a former commander of
SPACECOM, explained, the United States "will engage terrestrial targets someday-ships, airplanes, land targets-from space. We will
engage targets in space, from space . . . . [The missions are] already assigned, and we've written the concepts of operations."7
SPACECOM also claims that the United States must establish a military presence in space in 8
order to preempt possible efforts by other nations to do so . Although this doctrine was once advanced
exclusively by SPACECOM, prominent civilian defense officials have endorsed the global engagement strategy, and have begun to
implement changes in Pentagon doctrine, organization, and budgets to move in that direction. The January 2001 Rumsfeld
Commission report on the management of U.S. space assets, produced by a study commission chaired by Donald Rumsfeld before
he became Secretary of Defense, signaled his strong support for the need to project force in space in order to counter presumed
threats to U.S. military security there.9 Although it stopped short of directly advocating space weapons, no one could miss the point.
In late September 2001, the QuadrennialDefense Review Report, a wide-ranging assessment of U.S defense policy, called for beefing
up military space surveillance, communications, and other applications of earth-orbiting spacecraft. It also underscored the need to
deny the use of space by adversaries, and to address U.S. vulnerabilities in space with aggressive development of space capabilities.
10 Most tellingly, the Department of Defense's Nuclear Posture Review, portions of which were leaked in March 2002, reportedly
advocates the use of space-based assets to enhance conventional and nuclear strike capabilities.' In October 2002, SPACECOM
merged with the U.S. Strategic Command, which controls U.S. nuclear forces, to create a single entity responsible for early warning,
missile defense, and long-range strikes. 2 The Pentagon requested $1.6 billion dollars over FY 2003-2007 to develop space-based
lasers and kinetic kill vehicles to intercept and destroy ballistic missiles (and to destroy satellites). 13 Providing further evidence of
high-level support for the global engagement strategy, the current Bush administration's decision to withdraw from the 30-year-old
Anti-Ballistic Missile (ABM) Treaty 14 appeared to be less of a necessary move driven by technical demands of missile defense
testing (since much testing could be done within the terms of the treaty, and deployment of a feasible system is not imminent) than
a symbolic move to sweep away inconvenient legal obstacles to U.S. power 5 projection in space. This
vision of national
dominance, the rest of the world, and especially 6 China, contends, is incompatible with the established legal
regime in space.' The international community has for over forty years repeatedly reaffirmed
that space should be preserved for peaceful purposes, should be available to all, and should be
weapon-free. Hence the relevant options appear to be reduced to two: an active contest over
national superiority in space, or an elaborated legal regime that would undoubtedly be designed
to prevent decisive predominance in space by any one country, the United States in 17 particular.
A contest over national superiority in space could extinguish the explicit equal right to use space
that all nations enjoy, creating instead a de facto regime of control over access and use by the
first nation to successfully deploy weapons based in space or weapons on the ground that target
satellites. Given the immense value of outer space and its resources, other nations might develop their own
antisatellite weapons designed to break this monopoly . Countries that lack the capabilities to build such
weapons might purchase them. Space-based weapons would also generate instability due to the
incentives for preemptive attack that powerful but vulnerable weapons systems seem likely to
create. A more elaborated legal regime would be aimed at preventing destabilizing conflicts
over the use of space. The problem posed is how to balance the interests of the United States with those of the rest of the
world. The SPACECOM position, if seriously pursued, would pit the United States against everyone else, and the support of even
close allies could be in question. Equally if not more important, other significant interests of the United States in space would be
jeopardized if an extended battle over space superiority were to develop. Given the inherent vulnerability of space activities,
traditional military support activities (including space-tracking, early warning, communications, reconnaissance, weather monitoring,
and navigation) would be placed in jeopardy. The viability of commercial and scientific activities in space would come into serious
question as well. In a conflict, terrestrial components of space activities could become objects of attack, while attacks against
satellites could litter space with speeding debris that might rip into commercial satellites and space vehicles, disrupting commercial
and scientific activity and communications on the ground. Although SPACECOM and its supporters aggressively assert their views,
advocates of space weaponry may be in the minority, even in the Pentagon. As many observers recognize, the interests of the
United States in space are much broader than SPACECOM presents. U.S. testing and deployment of orbital weapons could make the
use of space for other military and commercial purposes more difficult. Many in the military, especially those involved in crucial
military support activities, are quietly aware of this, as are officials at the National Aeronautics and Space Administration (NASA) and
the International Space Station (ISS),20 and their supporters in Congress. Congressional support for space weapons programs is
sometimes difficult to assess, but does not appear to be deep or widespread.21 Serious questions remain as to whether the threats
to U.S. assets in space are really as great as SPACECOM argues, and whether, even if the threats were real, expensive and difficult
space-based weapons would be the most effective way to deal with them. In many cases, those wishing to hurt the United States
likely will find it much easier, and more effective, to attack terrestrial targets.22 Overall, the
risks brought on by a
competition for national dominance in space would ultimately be detrimental to the United
States. The United States is by far the nation most reliant on space for its military and economic well- being. It has an estimated
850 satellites, both military and commercial, in orbit-a number that is expected to increase substantially during the next ten years.23
Although this technological and financial edge in space will grow in the short term, the United States ultimately will see that
advantage diminish over time. Current U.S. space doctrine such as that articulated in Visionfor 2020 likely underestimates the speed
with which the U.S. advantage as a space power will erode (although SPACECOM advocates hope to preserve this advantage through
space domination).24 The
choice between a competition for national superiority and a strengthened
legal regime that preserves and balances the interests of all in space will have profound
consequences. If the United States aggressively moved weaponry into space, it would likely
provoke other nations to pursue countermeasures, with destabilizing consequences for global
and national security. In addition, by encouraging nations who do not currently have an interest in placing weapons in space
to compete directly and immediately with U.S. space-based assets, the United States would almost certainly guarantee the loss of
the advantages it seeks to protect. Although
an arms race in ASAT weapons is one of the dangers, the
threat currently of greatest concern to states such as China and Russia is the U.S. use of space
systems to augment its nuclear and conventional strategic strike capabilitie s. From the perspective of
these nations, the U.S. decision to expand strategic capabilities into space represents the collapse of the Cold War bargain of
strategic stability based on mutual vulnerability. A
military competition in space could thus invigorate a high-
tech arms race and renew emphasis on doctrines of nuclear warfare .25 Finally, a military
competition in space would largely extinguish the role of law in space in favor of a regime of
power. Despite the narrow organizational appeal of the latter to SPACECOM, the much broader interests of the United States in
space lie in the promotion of the rule of law. The United States has a long history of supporting the rule of
law both at home and in global affairs-in the latter case, promoting the development of rules that would secure U.S.
interests in an interdependent world.26 When presented with the choice , it is likely that most users of
space-including the satellite communications industry, those involved in military support operations, and the scientific community,
including NASA-would prefer the more stable protection provided by the rule of law rather than the
more uncertain and potentially disruptive protection of untested and complex weapons system s.
In sum, the United States and the international community have a strong interest in preventing a destabilizing military competition
in space through the timely negotiation of a more elaborated legal regime for space.

In the space domain, diplomatic arms control offers the most epistemologically
sound approach to breaking out of the security dilemma. It transforms mutual
vulnerability into common interest
Mutschler 13 – Max M. Mutschler, Researcher at the German Institute for International and
Security Affairs, Berlin, holds a doctoral degree from the Eberhard-Karls University Tübingen
(“Analyzing Arms Control in Space,” Chapter 6 in Arms Control in Space, Palgrave Macmillan,
Available through Springer)

The knowledge-based approach helps to solve the puzzles of the other two approaches. According to this
approach, states preferences are not exogenously given but are the result of internal processes of

interpretation. Which option is better for national security: unhindered arms dynamic or arms control? How this question is answered depends to a considerable extent upon the interpretation
of the situation at hand. Here, the factors power and interests come back in . How do states evaluate power

relationships? Are they self-centered and focus on their own strength with regard to a certain
weapon technology, or do they consider the consequences of their actions in an interdependent
world? Naturally, in the second case, the chances for preventive arms control are much higher than in the first case. In addition, several other questions are
related to this rather general issue: Does verification have to be waterproof or is a high
probability of detection sufficient? Do all sides have to possess the same standard in all fields of
weapon technology, or is it acceptable to make package-deals that account for those differences? How these questions are answered can
be influenced decisively by national and transnational political processes of learning.

an arms race has negative consequences for national security . It is this


The most important thing that must be learned is that

learning that transforms the Deadlock situation into a Prisoner’s Dilemma, thereby making
cooperation desirable in the first place. It seems plausible that cases of future weapon systems that are still in the process of development, at first resemble the
situation structure of a Deadlock game. Each state that is developing the respective weapon believes that the

development improves its national security, because it brings additional military capability.
Usually, the effects of the new weapons are unclear . In particular, because of the secrecy surrounding weapon development, states do not know how
others will and can react. Particularly in cases of technological asymmetry, the more advanced states can be expected to think that arming is better than arms control.

If a government believes that its weapon systems are superior to those of its rival, or that the strategy that will guide their use is superior, it may prefer mutual defection to mutual cooperation. (Downs et al. 1985:
122) The weaker states, while officially calling for arms control, might seek to exploit the weaknesses that often are the downside of asymmetric power. None of them think ahead and consider the negative

Learning about these consequences transforms the Deadlock


consequences of the inherent interdependence for its national security.

situation into a Prisoner’s Dilemma , thereby enabling cooperation. While the maximization of national security is the main goal of
policy-makers, they reinterpret their situation on the basis of new knowledge that points out the inconsistency between the goal – security – and the means – unilateral armament. While doubts regarding the
technical feasibility of a weapon system might prolong the process of R&D and delay procurement and deployment; it seems to be strategic reasoning, by doubting the added value for security of the weapon

system that makes states agree to preventive arms control. A closer look at the respective internal debates has shown that while
plenty of arguments against ABMs on the basis of technical shortcomings of whatever sort were
made by the critics, these arguments did not make the decisive points against ABMs. Right from the start,
especially in the U.S., there were massive doubts about whether ABM could ever work properly. These doubts have played an important role in

providing arguments for the critics to postpone the production and deployment of the systems ,
but they were not strong enough to make the case for arms control. Instead, it was the knowledge about the interdependent nature

of the arms dynamic and the impossibility of achieving security by unilateral armament which
convinced the top political decision-makers to seek an arms control solution .

Such a learning process transforms the Deadlock game , where the players prefer mutual
defection over cooperation, into a Prisoner’s Dilemma situation, where they prefer cooperation over mutual defection. We have seen
such a learning process in the case of ABMs but not in the case of space weapons. This is the central difference between the two cases, which explains the different outcomes: a regime with regard to ABMs and a

nonregime in the case of space weapons. Both cases have situation structures that can be interpreted as resembling the situation structure of the Prisoner’s Dilemma; in both cases,
preventive arms control could provide balanced gains ; and in both cases, verification measures
would allow states to apply a strategy of reciprocal cooperation. The major difference between the two cases is a process of learning
about the negative security consequences of an arms race, which has happened in one case but not in the other. Again, it is not important, if the things states have learned match reality. The

decisive point is that a certain interpretation of reality has triumphed over another one.

Scenario analysis is pedagogically valuable – enhances creativity and self-


reflexivity, deconstructs cognitive biases and flawed ontological assumptions,
and enables the imagination and creation of alternative futures.
Barma et al. 16 – (May 2016, [Advance Publication Online on 11/6/15], Naazneen Barma, PhD in
Political Science from UC-Berkeley, Assistant Professor of National Security Affairs at the Naval
Postgraduate School, Brent Durbin, PhD in Political Science from UC-Berkeley, Professor of
Government at Smith College, Eric Lorber, JD from UPenn and PhD in Political Science from
Duke, Gibson, Dunn & Crutcher, Rachel Whitlark, PhD in Political Science from GWU, Post-
Doctoral Research Fellow with the Project on Managing the Atom and International Security
Program within the Belfer Center for Science and International Affairs at Harvard, “‘Imagine a
World in Which’: Using Scenarios in Political Science,” International Studies Perspectives 17 (2),
pp. 1-19,
http://www.naazneenbarma.com/uploads/2/9/6/9/29695681/using_scenarios_in_political_scie
nce_isp_2015.pdf)
**FYI if anyone is skeptical of Barma’s affiliation with the Naval Postgraduate School, it’s worth looking at her
publication history, which is deeply opposed to US hegemony and the existing liberal world order:

a) co-authored an article entitled “How Globalization Went Bad” that has this byline: “From terrorism to global
warming, the evils of globalization are more dangerous than ever before. What went wrong? The world
became dependent on a single superpower. Only by correcting this imbalance can the world become a safer
place.” (http://cisac.fsi.stanford.edu/publications/how_globalization_went_bad)
b) most recent published scenario is entitled “World Without the West,” supports the a Non-Western
reinvention of the liberal order, and concludes that “This argument made a lot of people uncomfortable,
mostly because of an endemic and gross overestimation of the reach, depth and attractiveness of the
existing liberal order” (http://nationalinterest.org/feature/welcome-the-world-without-the-west-11651)

Over the past decade, the “cult of irrelevance” in political science scholarship has been lamented by a
growing chorus (Putnam 2003; Nye 2009; Walt 2009). Prominent scholars of international affairs have diagnosed the roots of the gap between
academia and policymaking, made the case for why political science research is valuable for policymaking , and offered a

number of ideas for enhancing the policy relevance of scholarship in international relations and comparative politics (Walt 2005,2011; Mead 2010; Van Evera 2010; Jentleson and Ratner 2011; Gallucci 2012;
several initiatives have been formed in the attempt to “bridge the gap.”2 Many of the specific efforts put in
Avey and Desch 2014). Building on these insights,

place by these projects focus on providing scholars with the skills, platforms, and networks to better

communicate the findings and implications of their research to the policymaking community, a necessary and worthwhile
objective for a field in which theoretical debates, methodological training, and publishing norms tend more and more toward the abstract and esoteric.

Yet enhancing communication between scholars and policymakers is only one component of bridging the gap between international affairs theory
and practice. Another crucial component of this bridge is the generation of substantive research programs that are

actually policy relevant—a challenge to which less concerted attention has been paid. The dual challenges of bridging the gap are especially acute for graduate students, a particular
irony since many enter the discipline with the explicit hope of informing policy. In a field that has an admirable devotion to pedagogical self-

reflection, strikingly little attention is paid to techniques for generating policy-relevant ideas for
dissertation and other research topics. Although numerous articles and conference workshops are devoted to the importance of experiential and problem-based learning, especially through
techniques of simulation that emulate policymaking processes (Loggins 2009; Butcher 2012; Glasgow 2012; Rothman 2012; DiCicco 2014), little has been written about the use of such techniques for generating
and developing innovative research ideas.

This article outlines an experiential and problem-based approach to developing a political science
research program using scenario analysis. It focuses especially on illuminating the research generation and pedagogical benefits of this technique by describing the use of
scenarios in the annual New Era Foreign Policy Conference (NEFPC), which brings together doctoral students of international and comparative affairs who share a demonstrated interest in policy-relevant
scholarship.3 In the introductory section, the article outlines the practice of scenario analysis and considers the utility of the technique in political science. We argue that scenario analysis should be viewed as a
tool to stimulate problem-based learning for doctoral students and discuss the broader scholarly benefits of using scenarios to help generate research ideas. The second section details the manner in which NEFPC
deploys scenario analysis. The third section reflects upon some of the concrete scholarly benefits that have been realized from the scenario format. The fourth section offers insights on the pedagogical potential
associated with using scenarios in the classroom across levels of study. A brief conclusion reflects on the importance of developing specific techniques to aid those who wish to generate political science
scholarship of relevance to the policy world.

What Are Scenarios and Why Use Them in Political Science?

Scenario analysis is perceived most commonly as a technique for examining the robustness of strategy. It can immerse decision makers in future
states that go beyond conventional extrapolations of current trends, preparing them to take
advantage of unexpected opportunities and to protect themselves from adverse exogenous
shocks. The global petroleum company Shell, a pioneer of the technique, characterizes scenario analysis as the art of considering “what if” questions about possible future worlds. Scenario
analysis is thus typically seen as serving the purposes of corporate planning or as a policy tool to
be used in combination with simulations of decision making. Yet scenario analysis is not inherently limited to these uses . This

section provides a brief overview of the practice of scenario analysis and the motivations underpinning its uses. It then makes a case for the utility of the
technique for political science scholarship and describes how the scenarios deployed at NEFPC were created.
The Art of Scenario Analysis

We characterize scenario analysis as the art of juxtaposing current trends in unexpected


combinations in order to articulate surprising and yet plausible futures, often referred to as
“alternative worlds.” Scenarios are thus explicitly not forecasts or projections based on linear
extrapolations of contemporary patterns, and they are not hypothesis-based expert
predictions. Nor should they be equated with simulations, which are best characterized as functional
representations of real institutions or decision-making processes (Asal 2005). Instead, they are
depictions of possible future states of the world , offered together with a narrative of the driving
causal forces and potential exogenous shocks that could lead to those futures. Good scenarios thus rely on explicit causal propositions that, independent of
one another, are plausible—yet, when combined, suggest surprising and sometimes controversial future worlds. For example, few predicted the dramatic fall in oil prices toward the end of 2014. Yet independent
driving forces, such as the shale gas revolution in the United States, China’s slowing economic growth, and declining conflict in major Middle Eastern oil producers such as Libya, were all recognized secular trends
that—combined with OPEC’s decision not to take concerted action as prices began to decline—came together in an unexpected way.

While scenario analysis played a role in war gaming and strategic planning during the Cold War, the real antecedents of the contemporary practice are found in corporate futures studies of the late 1960s and early
1970s (Raskin et al. 2005). Scenario analysis was essentially initiated at Royal Dutch Shell in 1965, with the realization that the usual forecasting techniques and models were not capturing the rapidly changing
environment in which the company operated (Wack 1985; Schwartz 1991). In particular, it had become evident that straight-line extrapolations of past global trends were inadequate for anticipating the evolving
business environment. Shell-style scenario planning “helped break the habit, ingrained in most corporate planning, of assuming that the future will look much like the present” (Wilkinson and Kupers 2013, 4).
Using scenario thinking, Shell anticipated the possibility of two Arab-induced oil shocks in the 1970s and hence was able to position itself for major disruptions in the global petroleum sector.

Building on its corporate roots, scenario analysis has become a standard policymaking tool. For example, the Project on Forward Engagement advocates linking systematic foresight, which it defines as the
disciplined analysis of alternative futures, to planning and feedback loops to better equip the United States to meet contemporary governance challenges (Fuerth 2011). Another prominent application of scenario
thinking is found in the National Intelligence Council’s series of Global Trends reports, issued every four years to aid policymakers in anticipating and planning for future challenges. These reports present a handful
of “alternative worlds” approximately twenty years into the future, carefully constructed on the basis of emerging global trends, risks, and opportunities, and intended to stimulate thinking about geopolitical
change and its effects.4 As with corporate scenario analysis, the technique can be used in foreign policymaking for long-range general planning purposes as well as for anticipating and coping with more narrow
and immediate challenges. An example of the latter is the German Marshall Fund’s EuroFutures project, which uses four scenarios to map the potential consequences of the Euro-area financial crisis (German
Marshall Fund 2013).
Several features make scenario analysis particularly useful for policymaking.5 Long-term global
trends across a number of different realms —social, technological, environmental, economic, and political—combine in often-
unexpected ways to produce unforeseen challenges . Yet the ability of decision makers to
imagine, let alone prepare for, discontinuities in the policy realm is constrained by their existing mental
models and maps. This limitation is exacerbated by well-known cognitive bias tendencies such
as groupthink and confirmation bias (Jervis 1976; Janis 1982; Tetlock 2005). The power of scenarios lies in their ability
to help individuals break out of conventional modes of thinking and analysis by introducing
unusual combinations of trends and deliberate discontinuities in narratives about the future .
Imagining alternative future worlds through a structured analytical process enables
policymakers to envision and thereby adapt to something altogether different from the
known present.
Designing Scenarios for Political Science Inquiry

Scenarios
The characteristics of scenario analysis that commend its use to policymakers also make it well suited to helping political scientists generate and develop policy-relevant research programs.

are essentially textured, plausible, and relevant stories that help us imagine how the future political-
economic world could be different from the past in a manner that highlights policy challenges and opportunities. For example, terrorist organizations are a known threat that
have captured the attention of the policy community, yet our responses to them tend to be linear and reactive. Scenarios that explore how seemingly unrelated vectors of change—the rise of a new peer
competitor in the East that diverts strategic attention, volatile commodity prices that empower and disempower various state and nonstate actors in surprising ways, and the destabilizing effects of climate change
or infectious disease pandemics—can be useful for illuminating the nature and limits of the terrorist threat in ways that may be missed by a narrower focus on recognized states and groups. By illuminating the
potential strategic significance of specific and yet poorly understood opportunities and threats, scenario analysis helps to identify crucial gaps in our collective understanding of global politicaleconomic trends and

Very simply, scenario analysis can


dynamics. The notion of “exogeneity”—so prevalent in social science scholarship—applies to models of reality, not to reality itself.

throw into sharp relief often-overlooked yet pressing questions in international affairs that
demand focused investigation.

Scenarios thus offer, in principle, an innovative tool for developing a political science research agenda . In
practice, achieving this objective requires careful tailoring of the approach . The specific scenario analysis technique we outline below
was designed and refined to provide a structured experiential process for generating problem-based research questions with contemporary international policy relevance.6 The first step in the process of creating
the scenario set described here was to identify important causal forces in contemporary global affairs. Consensus was not the goal; on the contrary, some of these causal statements represented competing
theories about global change (e.g., a resurgence of the nation-state vs. border-evading globalizing forces). A major principle underpinning the transformation of these causal drivers into possible future worlds was
to “simplify, then exaggerate” them, before fleshing out the emerging story with more details.7 Thus, the contours of the future world were drawn first in the scenario, with details about the possible pathways to
that point filled in second. It is entirely possible, indeed probable, that some of the causal claims that turned into parts of scenarios were exaggerated so much as to be implausible, and that an unavoidable degree
of bias or our own form of groupthink went into construction of the scenarios. One of the great strengths of scenario analysis, however, is that the scenario discussions themselves, as described below, lay bare
these especially implausible claims and systematic biases.8

An explicit methodological approach underlies the written scenarios themselves as well as the analytical process around them—that of case-centered, structured, focused comparison, intended especially to shed

The use of scenarios is similar to counterfactual analysis in that it


light on new causal mechanisms (George and Bennett 2005).

modifies certain variables in a given situation in order to analyze the resulting effects (Fearon 1991).
Whereas counterfactuals are traditionally retrospective in nature and explore events that did not actually occur in the context of known history,
our scenarios are deliberately forward-looking and are designed to explore potential futures that could
unfold. As such, counterfactual analysis is especially well suited to identifying how individual events might expand or shift the “funnel of choices” available to political actors and thus lead to different historical
outcomes (Nye 2005, 68–69), while forward-looking scenario analysis can better illuminate surprising intersections and sociopolitical dynamics without the perceptual constraints imposed by fine-grained historical

We see scenarios as a complementary resource for exploring these dynamics in


knowledge.

international affairs, rather than as a replacement for counterfactual analysis, historical case studies, or other methodological tools.
In the scenario process developed for NEFPC, three distinct scenarios are employed, acting as cases for analytical comparison. Each scenario, as detailed below, includes a set of explicit “driving forces” which
represent hypotheses about causal mechanisms worth investigating in evolving international affairs. The scenario analysis process itself employs templates (discussed further below) to serve as a graphical
representation of a structured, focused investigation and thereby as the research tool for conducting case-centered comparative analysis (George and Bennett 2005). In essence, these templates articulate key
observable implications within the alternative worlds of the scenarios and serve as a framework for capturing the data that emerge (King, Keohane, and Verba 1994). Finally, this structured, focused comparison
serves as the basis for the cross-case session emerging from the scenario analysis that leads directly to the articulation of new research agendas.

The scenario process described here has thus been carefully designed to offer some guidance to
policy-oriented graduate students who are otherwise left to the relatively unstructured norms by
which political science dissertation ideas are typically developed. The initial articulation of a dissertation project is generally an idiosyncratic and
personal undertaking (Useem 1997; Rothman 2008), whereby students might choose topics based on their coursework, their own previous policy exposure, or the topics studied by their advisors. Research
agendas are thus typically developed by looking for “puzzles” in existing research programs (Kuhn 1996). Doctoral students also, understandably, often choose topics that are particularly amenable to garnering
research funding. Conventional grant programs typically base their funding priorities on extrapolations from what has been important in the recent past—leading to, for example, the prevalence of Japan and
Soviet studies in the mid-1980s or terrorism studies in the 2000s—in the absence of any alternative method for identifying questions of likely future significance.

The scenario approach to generating research ideas is grounded in the belief that these
traditional approaches can be complemented by identifying questions likely to be of great
empirical importance in the real world, even if these do not appear as puzzles in existing research
programs or as clear extrapolations from past events. The scenarios analyzed at NEFPC envision alternative
worlds that could develop in the medium (five to seven year) term and are designed to tease
out issues scholars and policymakers may encounter in the relatively near future so that they
can begin thinking critically about them now. This timeframe offers a period distant enough
from the present as to avoid falling into current events analysis, but not so far into the future as
to seem like science fiction. In imagining the worlds in which these scenarios might come to pass, participants learn strategies for
avoiding failures of creativity and for overturning the assumptions that prevent scholars and
analysts from anticipating and understanding the pivotal junctures that arise in international
affairs.

The security dilemma is a robust and intellectually sound explanation of conflict


escalation
Jacobsen and Halvorsen 18 – (Jo, Associate Professor at the Norwegian University of
Science and Technology; Thomas, Senior Scientist at SINTEF ( Stiftelsen for industriell og teknisk
forskning) Technology and Society; 5/22/18; The Durability of the Security Dilemma: An
Empirical Investigation of Action–Reaction Dynamics in States’ Military Spending (1988–2014);
doi: 10.1093/cjip/poy007)

The security-dilemma model, for its part, rests on a spiral logic that highlights the self-defeating—tragic—
properties of security-seeking in an anarchic world ;15 that is to say, a world, ‘where one state’s
attempts to increase its security appear threatening to others and provoke an unnecessary
conflict’.16 States seek survival and security, and as they cannot be certain of the intentions of
others, military capabilities become the ultimate means of protection . But here, suspicion and fear are
mutual, resulting in a cyclical pattern: one state increases its arms; the other, fearing that the
arms build-up may rest on malign intentions, follows suit ; the first reacts to this; the second reacts to the first’s reaction, and so on. Both
states are pure, defensively-minded security-seekers—but none can afford to trust that the other
is of this type.
Anarchy, Tragedy, and the Security Dilemma

The concept of the security dilemma thus catches, ‘the unfortunate fact that policies designed
to increase the state’s security often have the effect of decreasing the other’s securit y’.17 States
accumulate power for defence, but considering that, ‘no state can know that the power
accumulation of others is defensively-motivated only, each must assume that it might be
intended for attack. Consequently, each party’s power increments are matched by the others, and all
wind up with no more security than when the vicious cycle began’.18 Such tragic spirals, ‘between
states that want nothing more than to preserve the status quo’19 represent, according to some, ‘the
quintessential dilemma in international politics’.20
It was John Herz21 who originally introduced the term, lucidly capturing the key elements on which later scholars—notably Herbert Butterfield,22 Robert Jervis, and Charles Glaser—elaborate. The

security-dilemma logic has since been used to explain, inter alia, the security environment in East
Asia;23 World War I;24 the onset and continuation of the Cold War ;25 ethnic conflict;26 alliance
politics;27 and US ballistic missile defences and Russian countermoves .28
For Herz, it all begins with the structure of the system—of any system without a higher authority. In such an anarchic system, he writes, what arises is a,
‘security dilemma’ of men, or groups, or their leaders. Groups or individuals living in such a constellation must be, and usually are, concerned about their security from being attacked, subjected, dominated, or
annihilated by other groups and individuals. Striving to attain security from such attack, they are driven to acquire more and more power in order to escape the impact of the power of others. This, in turn, renders
the others more insecure and compels them to prepare for the worst. Since none can ever feel entirely secure in such a world of competing units, power competition ensues, and the vicious cycle of security and
power accumulation is on.29

The dilemma is a structural one. It follows not from characteristics of states or individuals; it is
rather based at Kenneth Waltz’s third level of analysis,30 arising from the lack of a supranational
sovereign—that is, from anarchy.31 This is a self-help, competitive system wherein actors or states are constrained with respect to their freedom of manoeuvre.
Security and survival being their fundamental goals, states are apt to err on the side of caution
in their security policies, constantly striving either to improve or to keep their power position
vis-à-vis others. For not doing so, considering the possibility that the motives or intentions of those others might not be benevolent, involves the risk
of being exploited.

the fear with which it is associated, ‘most strongly drives the security dilemma’. 32 Its command generates
This risk, and

efforts to maximise security by augmenting relative power. But when two (or more) states
simultaneously act according to this logic , both (all) will at the very least wind up no better off in terms of
security, and bearing the added costs that go with security competition and arms races .33 Indeed, security
should be reduced all around, because the vicious spiral enhances mutual suspicion and tensions.34 Worse still, if military

technology and prevailing strategies are such that striking first is rationally tempting, the
security dilemma mechanism can, by itself, trigger war.35

The security dilemma is a tragic dilemma, in the sense thatstates do not seek to become engaged in conflicts and vicious spirals ;
instead, the structural constraints under which they operate induce or compel them to undertake

actions that are in reality self-defeating .36 Mutual security is preferred, but security competition ensues as an unintended consequence of moves by, ‘decision
makers finding themselves in a predicament that is not of their own making’.37 The motives or intentions of actors play no necessary role in

the tragedy. Others’ intentions cannot be known for certain—and their future intentions are
impossible to predict. This means that even in a world consisting solely of security-seeking or
status quo-oriented states—as opposed to power-seeking, ‘revisionist’, or ‘greedy’ ones—fear and uncertainty prevail, as does the
security dilemma. As Robert Jervis observes, this fear and uncertainty stem not from any ‘limitations on rationality imposed by human psychology nor in a flaw in human nature, but in a
correct appreciation of the consequences of living in a Hobbesian state of nature’.38 The build-up of military capabilities , therefore, can be viewed as a

prudent response to an uncertain future (or present) in which worst-case scenario planning constitutes
insurance against threats to one’s security or survival .39

each state or player, under conditions of


This fits with the Prisoner’s Dilemma analogy, which Robert Jervis in particular has pondered and elaborated:40

imperfect information, rationally follows a strategy of ‘defection’, as opposed to one of


‘cooperation’, to avoid the ‘sucker’s payoff’. Both (or all) having done so, their interaction produces a Pareto suboptimal outcome, for both (all) would have
preferred mutual cooperation to reciprocal defection. But the conflict outcome—the game's ‘solution’—still has the character of a Nash equilibrium, which follows rationally from the game’s properties. Again, it
is fundamentally structure (anarchy) coupled with the inescapable factor of information
deficiency that compels such a tragic outcome , even if the players’ preference orderings are
overwhelmingly status-quo inclined, in which case the game is not a Prisoners’ Dilemma but a Stag Hunt, wherein mutual cooperation is preferred even to the unilateral
defection. Yet, so long as the players are uncertain about which game they are actually participants of, defection should be the strategy of choice, and

conflict should therefore ensue.

states still try to estimate others’ motives, and in doing so are apt to pay
The ubiquitous uncertainty notwithstanding,

heed to the behaviour of potential security competitors , not least to their military spending and
posture.41 It is exactly here that the delicate balancing between security-enhancing and self-defeating behaviour commences. This constitutes a dilemma in itself. If a given state
has an incentive to signal benign motives to its adversary, it will (depending on the offence–defence balance, which is described later)
avoid augmenting military capabilities for fear the other will interpret this as signalling malign
intentions. At the same time, though, such a decision will necessarily put the former in a vulnerable position,
which it can scarcely afford given the prominence of security concerns under the perilousness of
anarchy.42 Contrarily, if the state instead increases its military spending, it risks signalling malign
intentions, in which case the second state would rationally react by doing the same.
Most states facing this situation would probably be inclined to settle for the ‘least-bad’ option, which entails the sacrifice of revealing their true, benign motives on the altar of military capabilities.43 However, this

the former’s arms build-up would signal both


still constitutes a quandary for the second state that would ultimately make it ‘doubly insecure’.44 That is to say,

enhanced military capacity and malevolent intentions . Consequently the second state, for its part, would be ill advised to let a potentially
‘greedy’45 or ‘imperialist’46 state gain an unfettered advantage with regard to capabilities. At core here is the reluctance or inability—out of fear,

uncertainty or risk aversion—to perceive the situation as a security dilemma, even when that is
what it really is. Two states, both of which are status-quo oriented, may thus end up, ‘in a
relationship of higher conflict than is required by the objective situation’.

Root cause explanations of International Relations don’t exist – methodological


pluralism is necessary to reclaim IR as emancipatory praxis and avoid endless
political violence.
Bleiker 14 – (6/17, Roland, Professor of International Relations at the University of Queensland,
“International Theory Between Reification and Self-Reflective Critique,” International Studies
Review, Volume 16, Issue 2, pages 325–327)
This book is part of an increasing trend of scholarly works that have embraced poststructural critique but want to ground it in more positive political foundations, while retaining a reluctance to return to the
positivist tendencies that implicitly underpin much of constructivist research. The path that Daniel Levine has carved out is innovative, sophisticated, and convincing. A superb scholarly achievement.

the key challenge in international relations (IR) scholarship is what he calls “unchecked reification”: the widespread and
For Levine,

dangerous process of forgetting “the distinction between theoretical concepts and the real-
world things they mean to describe or to which they refer” (p. 15). The dangers are real, Levine stresses, because IR deals
with some of the most difficult issues, from genocides to war. Upholding one subjective position
without critical scrutiny can thus have far-reaching consequences. Following Theodor Adorno—who is the key theoretical influence on this
book—Levine takes a post-positive position and assumes that the world cannot be known outside of our human perceptions and the values that are inevitably intertwined with them. His ultimate goal is to
overcome reification, or, to be more precise, to recognize it as an inevitable aspect of thought so that its dangerous consequences can be mitigated.

Levine proceeds in three stages: First he reviews several decades of IR theories to resurrect critical moments when scholars displayed an acute awareness of the dangers of reification. He refreshingly
breaks down distinctions between conventional and progressive scholarship, for he detects self-reflective and critical moments in scholars that are usually

associated with straightforward positivist positions (such as E.H. Carr, Hans Morgenthau, or Graham Allison). But Levine also shows how these moments of self-reflexivity
never lasted long and were driven out by the compulsion to offer systematic and scientific knowledge.

outlines why IR scholars regularly closed down critique. Here, he points to a range of factors and phenomena, from peer review processes to the speed at which
The second stage of Levine's inquiry

academics are meant to publish. And here too, he eschews conventional wisdom, showing that work conducted in the wake of the third debate, while explicitly post-

positivist and critiquing the reifying tendencies of existing IR scholarship, often lacked critical self-awareness. As a result, Levine believes that
many of the respective authors failed to appreciate sufficiently that “reification is a consequence of all

thinking—including itself” (p. 68).

sustainable critique”: a form of self-reflection that can counter the


The third objective of Levine's book is also the most interesting one. Here, he outlines the path toward what he calls “

dangers of reification. Critique, for him, is not just something that is directed outwards, against particular theories or

theorists. It is also inward-oriented, ongoing, and sensitive to the “limitations of thought itself” (p.
12).

The challenges that such a sustainable critique faces are formidable. Two stand out: First, if the natural tendency to forget the origins and values of our concepts are as strong as Levine and other Adorno-inspired
theorists believe they are, then how can we actually recognize our own reifying tendencies? Are we not all inevitably and subconsciously caught in a web of meanings from which we cannot escape? Second, if one
constantly questions one's own perspective, does one not fall into a relativism that loses the ability to establish the kind of stable foundations that are necessary for political action? Adorno has, of course, been
critiqued as relentlessly negative, even by his second-generation Frankfurt School successors (from Jürgen Habermas to his IR interpreters, such as Andrew Linklater and Ken Booth).

He starts off with depicting


The response that Levine has to these two sets of legitimate criticisms are, in my view, both convincing and useful at a practical level.

reification not as a flaw that is meant to be expunged, but as an a priori condition for
scholarship. The challenge then is not to let it go unchecked.
Methodological pluralism lies at the heart of Levine's sustainable critique. He borrows from what Adorno calls a
“constellation”: an attempt to juxtapose, rather than integrate, different perspectives. It is in this spirit that Levine advocates multiple methods to understand the

same event or phenomena. He writes of the need to validate “multiple and mutually
incompatible ways of seeing” (p. 63, see also pp. 101–102). In this model, a scholar oscillates back and forth between
different methods and paradigms, trying to understand the event in question from multiple
perspectives. No single method can ever adequately represent the event or should gain the
upper hand. But each should, in a way, recognize and capture details or perspectives that the others
cannot (p. 102). In practical terms, this means combining a range of methods even when—or, rather,
precisely when—they are deemed incompatible. They can range from poststructual
deconstruction to the tools pioneered and championed by positivist social sciences.

The benefit of such a methodological polyphony is not just the opportunity to bring out nuances
and new perspectives. Once the false hope of a smooth synthesis has been abandoned, the very
incompatibility of the respective perspectives can then be used to identify the reifying
tendencies in each of them. For Levine, this is how reification may be “checked at the source” and this is how a
“critically reflexive moment might thus be rendered sustainable” (p. 103). It is in this sense that Levine's
approach is not really post-foundational but , rather, an attempt to “balance foundationalisms against
one another” (p. 14). There are strong parallels here with arguments advanced by assemblage thinking and complexity theory—links that could have been explored in more detail.

Outer space policy is an essential subject of debate---space militarization issues


open ground for profound questions about the nature of society, politics and
humanity, as well as possibilities for ontological transformation of the
political---the aff’s analysis breaks through the totalizing terms of status quo
debates
Natlie Bormann 9, Professor of Politics at Northeastern University; with Michael Sheehan, IR
Professor; 2009, “Introduction,” in Securing Outer Space: International Relations Theory and the
Politics of Space, p. 10-20

our thinking about socio-political, economic and military-related issues were defined,
For fifty years, much of

shaped and driven by the Cold War and the centrality of a comfortable paradox — that of a bipolar nuclear confrontation. A decade and a half after the end of that
confrontation we are still deemed to be living in a period, the ‘post’—Cold War era, that is defined only in relation to the preceding one. And while there is a strong temptation ,

if not an expectation, for some scholars to adhere to these well-known and totalizing terms of the
debate, for others the past two generations have been animated by a different, and pervasive, intervention — the ‘space age’. The movement of humanity into space and the development of satellite
technology in retrospect may well appear as the defining characteristic of this period.

The fiftieth anniversary of the beginning of the space age was marked on 4 October 2007. It was on this day, in 1957, that the Soviet Union launched Sputnik 1, the first satellite to be placed in orbit. This dramatic

the space era, it also triggered a set of questions regarding the assumptions and effects
event not only ushered in

that were (and are) constitutive of this new endeavour : questions of the global, the international,
the political, the ethical, the technical, the scientific, humankind and modernity — to name but a
few. In what ways would these questions guide, alter and intervene with our activities in space?
But also, in what ways would the space age guide, alter and intervene with these questions?

concerns regarding the modes and kinds of space activities that we would be witnessing, and these
That day in October 1957 also marked the beginning of serious

concerns were dominated from the outset by the fact that the first journey into space was
accompanied by — if not entirely driven by — the Cold War arms race. The initial steps in the exploration
of space were inexorably linked with pressures to militarize and securitize this new dimension . As a
raised profound
geographical realm that had hitherto been pristine in relation to mankind’s warlike history, this immediate tendency for space exploration to be led by military rationales

philosophical and political questions. What should the purpose of space activity be, and what
should it not be? And how would we approach, understand and distinguish between military activities, civilian ones, commercial ones, and so forth?

questions as to ‘what we bring to space’ as well as how space activities challenge us, and to what effects, seem
More than a half century later, the

ever more pressing. While the debate over some of the assumptions, modes and effects of the space age never truly abated, most of the contributors in this volume agree that there is sense of

urgency in raising concern, re-conceptualizing the modes of the debate, and engaging critically with the
limits and possibilities of the dimension of space vis-a-vis the political.

This sense of urgency reflects the revitalization of national space programmes, and particularly
that of the United States and China since the start of the twenty—first century. In January 2004, at NASA
headquarters, US President George W. Bush announced the need for a new vision for America’s civilian and scientific space programme. This call culminated in a Commission’s Report on Implementation of United
States Space Exploration Policy, which emphasized the fundamental role of space for US technological leadership, economic validity, and most importantly, security. While this certainly stimulated the debate over
the future direction of US space exploration, it has led many to express concern over the implicitly aggressive and ambitious endeavour of colonizing space in the form of calling upon the need for permanent
access to and presence in space. A critical eye has also been cast on the Commission’s endorsement of the privatization and commercialization of space and its support for implementing a far larger presence of
private industry in space operations.

at the forefront of the current debate on space activities are notions of its militarization and
Certainly also

securitization. The deployment of technologies with the aim to secure, safeguard, defend and control certain assets, innovations and activities in
space is presented to us as an inevitable and necessary development. It is argued that just as the development of reconnaissance
aircraft in the First World War led inexorably to the emergence of fighter aircraft to deny the enemy the ability to carry out such reconnaissance and then bombers to deliver weapons against targets that could be
identified and reached from the air, so too has the ‘multiplier effect’ on military capabilities of satellites encouraged calls for the acquisition of space—based capabilities to defend one’s own satellites and attack
those of adversaries, and in the longer term, to place weapons in space that could attack targets on Earth. Here, the Bush administration’s indication that it envisaged a prominent role for space—based Weapons
in the longer term as part of the controversial national missile defence system contributed to the atmosphere of controversy surrounding space policy.

As space has become crucial to, and utilized by, far more international actors, so the political implications of space activities have multiplied. The members of the European Space Agency have pursued space
development for economic, scientific and social reasons. Their model of international space cooperation has been seen as offering an example to other areas of the world, particularly in their desire to avoid
militarizing efforts. Yet even Europe has begun to develop military space capabilities, following a path that has already been pursued by other key states such as China and India, suggesting that there is an

How we conceptualize space has therefore become of


inevitability about the militarization, and perhaps ultimately the weaponization, of space.

fundamental moral, political and strategic importance.

Outer space challenges the political imagination as it has always challenged the human imagination in many other fields. For millennia people have looked up to
the stars and imagined it as the home of gods or the location of the afterlife. For centuries they have looked to it for answers about the physical nature of the universe and the place of mankind’s ancestral home

Space exploration is a driver of innovation, encouraging


within it. And for decades, it has been seen as the supreme test for advanced technology.

us to dream of what might be possible, to push back the boundaries of thought and to change
the nature of ontological realities by drawing on novel epistemologies . The physical exploration of the solar system through
the application of science and technology has been the visible demonstration of this.

The challenges that space poses for political theory are profound. If space is about the use of imagination, and the application of novel developments to create new possibilities for human progress, how has
political theory and political reality responded to this challenge? The answer, at least thus far, is both that it has changed everything, and that it has changed very little. For international law, most notably in the
Outer Space Treaty, the denial of territoriality and limitations on sovereignty beyond planet Earth offers a fundamental challenge to the way in which international relations has been conceptualized and

On the other hand, the dream of many, that humanity would leave behind its
operationalized in the modern era.

dark side as it entered space, has not been realized. For the most part, the exploration and
utilization of space has reflected, not challenged, the political patterns and impulses that
characterized twentieth—century politics and international relations. Propaganda, military
rivalry, economic competition and exploitation, North-South discrimination and so on have extended
their reach beyond the atmosphere. Industrialization and imperialism in the nineteenth century
helped produce powerful new social theories, as well as new philosophy, political ideologies
and conceptualizations of the meaning of politics and the nature of human destiny. The realities
of the space age demand novel social theories of the same order.
With the above in mind, this volume is undoubtedly driven by, and wants to speak to, the articulation of a number of controversial policies, contentious strategies and linguistic utilizations that promote space
activities in various ways under the rubric of exploration and innovation, militarization and weaponization, colonization and commercialization. However, while the contributions here were clearly prompted by
recent and current policies, the aim of this book is neither necessarily to explain these policies and discuss the merits of the space age, nor to offer more ‘workable’ solutions. Rather, it seeks to place them in a
broader theoretical perspective by attempting to achieve two objectives.

First, it explores ways in which we can articulate an understanding of that which precedes and informs these policies to begin with. The contributions here engage in a reading of space that traces the discourses of
space activities, and particularly their military dimension, exposes their meaning- producing practices, and unearths the narratives that give possibility to seeing particular space strategies as desirable, inevitable
and seamless. The authors also draw attention to the contexts within which activities ‘out there’ in space are always embedded, and which we need to remind ourselves of when contemplating space
developments. The contributions emphasize the fact that space policies resonate practices already central to the overall ‘earth—bound’ forging of foreign policies, security strategies, development, the ‘war on
terror’, globalization and so forth.
we can articulate an understanding of, and critically engage with, the effects of particular manifestations of
Second, the book suggests ways in which

space policies. While our conception of space is always constitutive of what already ‘is’, the projection
of activities in space (and their possibilities) similarly constitute, produce and shape socio-political relations
and activities on Earth. The contributions here reflect critically on questions of sovereignty; perceptions of time and space; modes of destruction, fighting and killing in, from and through
space; and relations of technology and ethics.
2AC
2AC Solvency
Alternative doesn’t solve—problematizing isn’t sufficient, only challenging the
way we think about security threats through concrete policy actions can
challenge traditional national security
Cole 12 (David COLE Law @ Georgetown ’12 “Confronting the Wizard of Oz: National Security, Expertise, and Secrecy”
CONNECTICUT LAW REVIEW 44 (5) p. 1629-1633)

Rana is right to focus our attention on the assumptions that frame modern Americans'
conceptions about national security, but his assessment raises three initial questions. First, it seems far from clear
that there ever was a "golden" era in which national security decisions were made by the
common man, or "the people themselves," as Larry Kramer might put it.8 Rana argues that neither Hobbes nor Locke would
support a worldview in which certain individuals are vested with superior access to the truth, and that faith in the superior abilities
of so-called "experts" is a phenomenon of the New Deal era. 9 While an increased faith in scientific solutions to social problems may
be a contributing factor in our current overreliance on experts,' 0 I doubt that national security matters were ever truly a matter of
widespread democratic deliberation. Rana notes that in the early days of the republic, every able-bodied man had to serve in the
militia, whereas today only a small (and largely disadvantaged) portion of society serves in the military." But serving in the militia
and making decisions about national security are two different matters. The early days of the Republic were at least as dominated by
"elites" as today. Rana points to no evidence that decisions about foreign affairs were any more democratic then than now. And, of
course, the nation as a whole was far less democratic, as the majority of its inhabitants could not vote at all. 12 Rather than moving
away from a golden age of democratic decision-making, it seems more likely that we have simply replaced one group of elites (the
aristocracy) with another (the experts). Second, to the extent that there
has been an epistemological shift with
respect to national security, it seems likely that it is at least in some measure a response to objective
conditions, not just an ideological development. If so, it's not clear that we can solve the problem
merely by "thinking differently" about national security. The world has, in fact, become more
interconnected and dangerous than it was when the Constitution was drafted . At our founding, the
oceans were a significant buffer against attacks, weapons were primitive, and travel over long distances was extremely arduous and
costly. The attacks of September 11, 2001, or anything like them, would have been inconceivable in the eighteenth or nineteenth
centuries. Small
groups of non-state actors can now inflict the kinds of attacks that once were the
exclusive province of states. But because such actors do not have the governance responsibilities that states have, they
are less susceptible to deterrence. The Internet makes information about dangerous weapons and civil vulnerabilities far more
readily available, airplane travel dramatically increases the potential range of a hostile actor, and it is not impossible that terrorists
could obtain and use nuclear, biological, or chemical weapons. 13 The knowledge necessary to monitor nuclear weapons, respond to
cyber warfare, develop technological defenses to technological threats, and gather intelligence is increasingly specialized. The
problem is not just how we think about security threats; it is also at least in part objectively based. Third,
deference to expertise is not always an error; sometimes it is a rational response to complexity. Expertise is generally
developed by devoting substantial time and attention to a particular set of problems . We cannot
possibly be experts in everything that concerns us. So I defer to my son on the remote control, to my wife on directions
(and so much else), to the plumber on my leaky faucet, to the electrician when the wiring starts to fail, to my doctor on my back
problems, and to my mutual fund manager on investments. I could develop more expertise in some of these areas, but that would
mean less time teaching, raising a family, writing, swimming, and listening to music. The same is true, in greater or lesser degrees,
for all of us. And it is true at the level of the national community, not only for national security, but for all sorts of matters. We defer
to the Environmental Protection Agency on environmental matters, to the Federal Reserve Board on monetary policy, to the
Department of Agriculture on how best to support farming, and to the Federal Aviation Administration and the Transportation
Security Administration on how best to make air travel safe. Specialization is not something unique to national
security. It is a rational response to an increasingly complex world in which we cannot possibly spend the
time necessary to gain mastery over all that affects our daily lives. If our increasing deference to experts on
national security issues is in part the result of objective circumstances, in part a rational
response to complexity, and not necessarily less "elitist" than earlier times, then it is not enough
to "think differently" about the issue. We may indeed need to question the extent to which we rely on experts, but
surely there
is a role for expertise when it comes to assessing threats to critical infrastructure,
devising ways to counter those threats, and deploying technology to secure us from
technology's threats. As challenging as it may be to adjust our epistemological framework, it seems likely that even if we
were able to sheer away all the unjustified deference to "expertise," we would still need to rely
in substantial measure on experts. The issue, in other words, is not whether to rely on experts, but
how to do so in a way that nonetheless retains some measure of self-government . The need for
specialists need not preclude democratic decision-making. Consider, for example, the model of adjudication. Trials involving
products liability, antitrust, patents, and a wide range of other issues typically rely heavily on experts.' 4 But critically, the decision is
not left to the experts. The decision rests with the jury or judge, neither of whom purports to be an expert. Experts testify, but do so
in a way that allows for adversarial testing and requires them to explain their conclusions to laypersons, who render judgment
informed, but not determined, by the expert testimony. Similarly, Congress routinely acts on matters over which its members are
not experts. Congress enacts laws governing a wide range of very complex issues, yet expertise is not a qualification for office.
Members of Congress, like many political appointees in the executive branch, listen to and consider the views of experts to inform
their decisions. Congress delegates initial consideration of most problems to committees, and by serving on those committees and
devoting time and attention to the problems within their ambit, members develop a certain amount of expertise themselves. They
may hire staff who have still greater expertise, and they hold hearings in which they invite testimony from still other experts. But at
the end of the day, the decisions about what laws should be passed are made by the Congress as a whole, not by the experts. A
similar process operates in the executive branch. The President and Vice-President generally need not be experts in any particular
field, and many of the cabinet members they appoint are not necessarily experts either. They are managers and policy makers. They
spend much of their day being briefed by people with more specialized expertise than they have. But at the end of the day, the
important decisions are made by politically accountable actors. Thus, deference
to experts need not preclude
independent or democratically accountable decision-making . The larger problem may be one that Rana notes
but does not sufficiently emphasize-an inordinate reliance on classified information and covert operations. 5 Secrecy is in many ways
the ultimate enemy of democracy in the national security realm. 16 As Judge Damon Keith has written, "democracy dies behind
closed doors.' ' 7 The experts in the intelligence community have the power to hide their decisions from external review and checks
by classifying the information they consider or the actions they take.18 Even if they do so in good faith, the inevitable result is that
their actions are increasingly insulated from scrutiny by others and immune from democratic checks. Virtually everyone who has had
access to classified information concedes that the system leads to massive over-classification. 19 Our overreliance on secrecy may
well be more central to the problem of inordinate deference than assumptions about the nature of knowledge regarding security.
And in any event, the problems are mutually reinforcing. The inaccessibility of the information the experts rely upon compels us to
defer to them because we lack sufficient grounds to question them. And that, in turn, may well make the experts more protective of
their information and more likely to classify their actions, decisions, and considerations.

For reference, their abstract


Howell and Richter-Montpetit 19 (Alison Howell Rutgers University, Newark, USA Melanie Richter-Montpetit
University of Sussex, UK , "Is securitization theory racist? Civilizationism, methodological whiteness, and antiblack thought in the
Copenhagen School" Security Dialogue 1–20 August 2019 /TheDON)

This article provides the first excavation of the foundational role of racist thought in Securitization Theory’s
(ST) conceptual and methodological project. We demonstrate that Copenhagen School ST is structured not only by Eurocentrism,
but also civilizationism, methodological Whiteness, and antiblack racism. Classic ST begins with a theory of ‘normal politics’ as
reasoned, civilized dialogue and securitization as a potential regression into a racially coded uncivilized ‘state of nature’. It
justifies this through a civilizationist history of the world that privileges Europe as the apex of
civilized ‘desecuritization’, sanitizing its violent (settler) colonial projects, and the racial violence of
normal liberal politics. It then constructs a methodologically White framework that uses speech act theory to locate
‘progress’ towards normal politics and the curbing of securitization in Europe. This methodological Whiteness produces normative
Whiteness, that is, the theory does not just describe desecuritization as European progress, it normatively asserts that becoming or
remaining like Europe is a moral imperative. Using ostensibly neutral terms, ST prioritizes order over justice, positioning the ST
theorist as the defender of (White) ‘civilized politics’ against (racialized) ‘primal anarchy’. Antiblackness
is a crucial
building block in ST’s conceptual edifice dividing security from politics: ST finds ‘primal anarchy’
especially in ‘Africa’, casting it as irrationally over-securitized, making it a foil to ‘civilized
politics’. We conclude by discussing whether other versions of ST emulate or dispense with the
racism of classic ST, and a discussion of whether the theory, or even just the concept of
securitization, can be recuperated.
2AC Afropessimism
Their thesis is wrong – anti-black racism is a contingent function of
institutionalized social power, which can be dismantled through the
transformation of these institutions
Gordon ‘17 (Lewis, professor of philosophy with affiliations in Judaic studies, Caribbean and
Latina/o studies, and Asian and Asian American studies at UCONN-Storrs, The Oxford Handbook
of Philosophy and Race, pp. 296-298)

Should the analysis remain at white and black, the world would appear more closed than it in
fact is. For one, simply being born black would bar the possibility of any legitimate appearance.
This is a position that has been taken by a growing group of theorists known as “Afro-
pessimists” (Wilderson 2010; Sexton 2011). Black for them is absolute “social death:’ it is outside
of relations. Missing from this view is; however, is at least what I argued in Bad Faith and
Antiblack Racism, which is that no human being is “really” any of these things; the claim itself is
a manifestation of mauvaise foi. The project of making people into such is one thing. People
actually becoming such is another. This is an observation Fanon also makes in his formulation of
the tone of nonbeing and his critique of Self—Other discourses in Peau noir, masques blancs
(Black Skin, White Masks). Fanon distinguishes between the zone of nonbeing (nonappearance
as human beings) and those of being. ‘The latter presumes a self-justified reality, which means it
does not call itself into question. The former faces the problem of illegitimate appearance
(Fanon 1952, chapters; Gordon 1999; AIcoir 2006; Yancy 2008). Thus, even the effort “to be” is
in conflict as the system in question presumes legitimate absence of certain groups. Yet,
paradoxically, the human being comes to the fore through emerging from being in the first
place. Thus, the assertion of being is also an effort to push the human being out of existence, so
to speak. The racial conflict is thus changed to an existential one in which an existential ontology
is posed against an ontology of being. Existential ontology pertains to human being, whereas
ontological being pertains to gods. This is why Fanon concludes that racism is also an attack
against human being, as it creates a world in which one set stands above others as gods and the
rest as below human. Where, in this formulation, stand human beings? The argument itself
gains some clarity with the etymology of “existence” which is from the Latin expression exsístere
(to stand out, to emerge -that is, to appear). Blacks thus face the paradox of existing (standing
out) as nonexistence (not standing out). The system of racism renders black appearance illicit.
This conundrum of racialized existence affects ethics and morals. Ethical relations are premised
on selves relating to another or others. The others must, however, appear as such, and they too,
manifest themselves as selves. Implicit in such others as other selves is the formalization of
ethical relations as equal. as found in the thought of Immanuel Kant and shifted in deference to
the other in that of Emmanuel Levinas, Racism, however, excludes certain groups from being
others and selves (if interpreted as being of a kind similar to the presumed legitimate selves).
Thus, the schema of racism is one in which the hegemonic group relates to its members as
selves and others, whereas the nonhegemonic groups are neither selves nor others. They, in
effect, could only be such in relation to each other. It is, in other words, a form of ontological
segregation as a condition of ethics and morals. The fight against racism, then, does not work as
a fight against being others or The Other. It is a fight against being nonothers. Fanon’s insight
demands an additional clarification. Racists should be distinguished from racism. Racists are
people who hold beliefs about the superiority and inferiority of certain groups of racially
designated people. Racism is the system of institutions and social norms that empower
individuals with such beliefs. Without that system, a racist would simply be an obnoxious,
whether overtly deprecating or patronizing, individual. With that system, racist points of view
affect the social world as reality. Without that system, racists ultimately become
inconsequential and, in a word, irrelevant beyond personal concerns of saving their souls from
unethical and immoral beliefs and choices. Fanon was concerned with racists in his capacity as a
psychiatrist (therapy, if necessary). but he was also concerned with racism as a philosopher,
social thinker, and revolutionary (Fanon 1959/1975). The latter, in other words, is a system,
from an antiracism perspective, in need of eradication. An objection to the Afro-pessimistic
assertion of blackness as social death could thus be raised from a Fanonian phenomenological
perspective: Why must the social world be premised on the attitudes and perspectives of
antiblack racists? Why don’t blacks among each other and other communities of color count as
a social perspective? And if the question of racism is a function of power, why not offer a study
of power, how it is gained and lost, instead of an assertion of its manifestations as ontological?
An additional problem with the Afro-pessimistic model is that its proponents treat “blackness”
as though it could exist independent of other categories. A quick examination of double
consciousness (Du Bois 1903)—a phenomenological concept if there ever were one by virtue of
the focus on forms of consciousness and, better, that of which one is conscious, that is,
intentionality would reveal why this would not work. Double conscious ness involves seeing
oneself from the perspective of another that deems one as negative (for example, the Afro-
pessimistic conception of blackness). That there is already another perspective makes the
subject who lives through double consciousness relational. Added is what Paget Henry (2005)
calls polemic, ted double consciousness and Nahum Chandler (2014, 6o—6i) calls the redoubled
gesture, which is the realization that the condemnation of negative meaning means that one
must not do what the Afro pessimist does. Seeing that that position is false moves one
dialectically forward into asking about the system that attempts to force one into such an
identity: This relational matter requires looking beyond blackness ironically in order to
understand blackness. This means moving from the conception of meaning as singular,
substance-based, fixed, and semantical into the grammar of how meaning is produced. Such
grammars, such as that of gender, emerge in interesting ways (Gordon 1999, 124—129; 1997,73
—74). However, as all human beings are manifestations of different dimensions of meaning, the
question of identity requires more than an intersecting model; otherwise there will simply be
one (a priori) normative outcome in every moment of inquiry: whoever manifests the maximum
manifestation of predetermined negative intersecting terms. That would in effect be an
essence before an existence indeed, before an actual event of harm. This observation emerges
as well with the Afro-pessimist model when one thinks of pessimism as the guiding attitude .
The existential phenomenological critique would be that optimism and pessimism are
symptomatic of the same attitude: a priori assertions on reality. Human existence is contingent
but not accidental, which means that the social world at hand is a manifestation of choices and
relationships in other words, human actions. Because human beings can only build the future
instead of it determining us, the task at hand, as phenomenology-oriented existentialists from
Beauvoir and Sartre to Fanon, William R. Jones, and this author have argued, depends on
commitment. This concern also pertains to the initial concerns about authenticity discourses
with which I began. One could only be pessimistic about an outcome, an activity. It is an act of
forecasting what could only be meaningful once actually performed. Similarly, one could only be
optimistic about the same. What however, if there were no way to know either? Here we come
to the foi element in mauvaise foi. Some actions are deontological, and if not that, they are at
least reflections of our commitments, our projects. Thus, the point of some actions is not about
their success or failure but whether we deem them worth doing . Taking responsibility for such
actions—bringing value to them— is opposed to another manifestation of mauvaise foi: the
spirit of seriousness.

Libidinal explanations of anti-Blackness are incorrect – it’s socially constructed,


not psychic, even though racism can be unconscious.
Hudis 15 – Peter Hudis, Professor of Humanities and Philosophy at Oakton Community College,
Ph.D. in Philosophy from Loyola University Chicago, 2015 (“Self and Other: The Dialectic of Black
Skin, White Masks,” Frantz Fanon: Philosopher of the Barricades, published by Pluto Press, no
day, ISBN: 9780745336251, pp. 35-37)

Fanon’s vantage point upon the world is his situated experience. He is trying to understand the
inner psychic life of racism, not provide an account of the structure of human existence as a
whole. Racism is not, of course, an integral part of the human psyche; it is a Social construct
that has a psychic impact. Any effort to comprehend social distress that accompanies racism by
reference to some a priori structure- be it the Oedipal Complex or the Collective Unconscious- is
doomed to failure. Carl Jung sought to deepen and go beyond Freud's approach by arguing that
the subconscious is grounded in a universal layer of the psyche- which he called "the collective
unconscious:' This refers to inherited patterns of thought that exist in all human minds,
regardless of specific culture or upbringing, and which manifest themselves in dreams, fairy
tales, and myths. Jung referred to these universal patterns as "archetypes:' It may seem, on a
superficial reading, that 1 Fanon is drawing from Jung, since he discusses how white people tend
to unconsciously assimilate views of blacks that are based on negative stereotypes. Even the
most "progressive" white tends to think of blacks a certain way (such as "emotional;' "physical,"
or / "aggressive"), even as they disavow any racist animus on their part. However, Fanon denies
that such collective delusions are part of a psychic structure; they are not permanent features
of the mind. They are habits acquired from a series of social and cultural impositions . While
they constitute a kind a collective unconscious on the part of many white people, they are not
grounded in any universal "archetype." The unconscious prejudices of whites do not derive from
genes or nature, nor do they derive from some form independent of culture or upbringing.
Fanon contends that Jung "confuses habit with instinct ." Fanon objects to Jung's "collective
unconscious" for the same reason that he rejects the notion of a black ontology. His
phenomenological approach brackets out ontological claims on both a social and psychological
level insofar as the examination of race and racism is concerned. He writes, "Neither Freud nor
Adler nor even the cosmic Jung took the black man into consideration in the course of his
research.” This does not mean that Fanon rejects their contributions tout court. He does not
deny the existence of the unconscious. He only denies that the inferiority complex of blacks
operates on an unconscious level. He does not reject the Oedipal Complex. He only denies that
it explains (especially in the West Indies) the proclivity of the black "slave" to mimic the values
of the white "master." And as seen from his positive remarks on Lacan's theory of the mirror
stage, he does not reject the idea of psychic structure. He only denies that it can substitute for
an historical understanding of the origin of neuroses .23 Fanon adopts a socio-genetic
approach to a study of the psyche because that is what is adequate for the object of his analysis.
For Fanon, it is the relationship between the socio-economic and psychological that is of critical
import. He makes it clear, insofar as the subject matter of his study is concerned, that the socio-
economic is first of all responsible for affective disorders: "First, economic. Then, internalization
or rather epidermalization of this inferiority."24 Fanon never misses an opportunity to remind
us that racism owes its origin to specific economic relations of domination- such as slavery,
colonialism, and the effort to coopt sections of the working class into serving the needs of
capital. It is hard to mistake the Marxist influence here. It does not follow, however, that what
comes first in the order of time has conceptual or strategic priority. The inferiority complex is
originally born from economic subjugation, but it takes on a life of its own and expresses itself in
terms that surpass the economic. Both sides of the problem-the socio-economic and
psychological-must be combatted in tandem: "The black man must wage the struggle on two
levels; whereas historically these levels are mutually dependent, any unilateral liberation is
flawed, and the worst mistake would be to believe their mutual dependence automatic:''5 On
these grounds he argues that the problem of racism cannot be solved on a psychological level .
It is not an "individual" problem; it is a social one. But neither can it be solved on a social level
that ores the psychological. It is small wonder that although his name never appears in the
book, Fanon was enamored of the work of Wilhelm Reich. This important Freudian-Marxist
would no doubt feel affinity with Fanon's comment, "Genuine disalienation will have been
achieved only when things, in the most materialist sense, have resumed their rightful place:'27

Their theory relies on the libidinal economy – that’s incorrect


Johnson 5. 2005, Adrian, PhD from SUNY-Stony Brook, Professor in the Department of
Philosophy at the University of New Mexico at Albuquerque and a faculty member at the Emory
Psychoanalytic Institute in Atlanta, “Time Driven: Metapsychology and the Splitting of the
Drive,” p. 340-1

Despite the apparent bleakness and antiutopianism of an assessment of human nature as being
perturbed by an irreducible inner antagonism , there is, surprisingly, what might be described as a liberating aspect
to this splitting of the drives. Since drives are essentially dysfunctional, subjects are able to act
otherwise than as would be dictated by instinctually compelled pursuits of gratification, satisfaction,
and pleasure. In fact, subjects are forced to be free, since, for such beings, the mandate of nature is forever
missing. Severed from a strictly biological master-program and saddled with a conflict-ridden, heterogeneous jumble of
contradictory impulses—impulses mediated by an inconsistent, unstable web of multiple representations ,

indicated by Lacan's “barring” of the Symbolic Other—the parlêtre has no choice but to bump up against the unnatural void of

its autonomy. The confrontation with this void is frequently avoided. The true extent of one's autonomy is, due to its sometimes-frightening implications, just as often
relegated to the shadows of the unconscious as those heteronomous factors secretly shaping conscious thought and behavior. The contradictions arising

from the conflicts internal to the libidinal economy mark the precise places where a freedom
transcending mundane materiality has a chance briefly to flash into effective existence; such
points of breakdown in the deterministic nexus of the drives clear the space for the sudden
emergence of something other than the smooth continuation of the default physical and
sociopsychical “run of things.” Moreover, if the drives were fully functional—and, hence, would not prompt a mobilization
of a series of defensive distancing mechanisms struggling to transcend this threatening corpo-Real— humans would be animalistic

automatons, namely, creatures of nature. The pain of a malfunctioning, internally conflicted


libidinal economy is a discomfort signaling a capacity to be an autonomous subject. This is a pain even
more essential to human autonomy than what Kant identifies as the guilt-inducing burden of duty and its corresponding pangs of anxious, awe-inspiring respect. Whereas Kant
treats the discomfort associated with duty as a symptom-effect of a transcendental freedom inherent to rational beings, the reverse might (also) be the case: Such freedom is
the symptom-effect of a discomfort inherent to libidinal beings. Completely “curing” individuals of this discomfort, even if it were possible, would be tantamount to divesting

As Lacan might phrase it, the split Trieb is the sinthome


them, whether they realize it or not, of an essential feature of their dignity as subjects.

of subjectivity proper , the source of a suffering that , were it to be entirely eliminated, would entail the

utter dissolution of subjectivity itself. Humanity is free precisely insofar as its pleasures are far
from perfection, insofar as its enjoyment is never absolute.

Anti-humanism is worse than humanism and humanism isn’t always bad


Lester 12 – (January 2012, Alan, Director of Interdisciplinary Research, Professor of Historical
Geography, and Co-Director of the Colonial and Postcolonial Studies Network, University of
Sussex, “Humanism, race and the colonial frontier,” Transactions of the Institute of British
Geographers, Volume 37, Issue 1, pages 132–148)

Anderson argues that it is not an issue of extending humanity to … negatively racialised people, but of
putting into question that from which such people have been excluded – that which, for liberal discourse, remains unproblematised. (2007, 199) I fear, however,
that if we direct attention away from histories of humanism’s failure to deal with difference and to
render that difference compatible with its fundamental universalism, and if we overlook its proponents’ failed attempts to combat dispossession,

murder and oppression; if our history of race is instead understood through a critique of humanity ’s conceptual separation
from nature, we dilute the political potency of universalism . Historically, it was not humanism that gave rise to

racial innatism, it was the specifically anti-humanist politics of settlers forging new social assemblages through

relations of violence on colonial frontiers . Settler communities became established social


assemblages in their own right specifically through the rejection of humanist interventions . Perhaps, as Edward
Said suggested, we can learn from the implementation of humanist universalism in practice ,
and insist on its potential to combat racism, and perhaps we can insist on the contemporary conceptual hybridisation of human–non-human entities too,
without necessarily abandoning all the precepts of humanism (Said 2004; Todorov 2002). We do not necessarily need to accord a specific value to the
human, separate from and above nature, in order to make a moral and political case for a fundamental human universalism that can be wielded

strategically against racial violence. Nineteenth century humanitarians’ universalism was fundamentally
conditioned by their belief that British culture stood at the apex of a hierarchical order of
civilisations. From the mid-nineteenth century through to the mid-twentieth century, this ethnocentrism produced what Lyotard describes as ‘the
flattening of differences, or the demand for a norm (“human nature”)’, that ‘carries with it its own forms of terror’ (cited Braun 2004, 1352). The intervention of Aboriginal Protection
demonstrates that humanist universalism has the potential to inflict such terror (it was the Protectorate of Aborigines Office reincarnated that was responsible, later in the nineteenth and twentieth centuries, for

But we must not


Aboriginal Australia’s Stolen Generation, and it was the assimilationist vision of the Protectors’ equivalents in Canada that led to the abuses of the Residential Schools system).

forget that humanism’s alternatives, founded upon principles of difference rather than
commonality, have the potential to do the same and even worse. In the nineteenth century,
Caribbean planters and then emigrant British settlers emphasised the multiplicity of the human
species, the absence of any universal ‘human nature’, the incorrigibility of difference, in their
upholding of biological determinism. Their assault on any notion of a fundamental commonality
among human beings has disconcerting points of intersection with the radical critique of
humanism today. The scientific argument of the nineteenth century that came closest to post-humanism’s insistence on the hybridity of humanity, promising to ‘close the
ontological gap between human and non-human animals’ (Day 2008, 49), was the evolutionary theory of biological descent associated with Darwin, and yet this theory was adopted in Aotearoa

New Zealand and other colonial sites precisely to legitimate the potential extinction of other, ‘weaker’ races in the
face of British colonisation on the grounds of the natural law of a struggle for survival (Stenhouse 1999). Both the upholding and the
rejection of human–nature binaries can thus result in racially oppressive actions, depending on the
contingent politics of specific social assemblages . Nineteenth century colonial humanitarians, inspired as they were by an irredeemably ethnocentric and religiously exclusive
form of universalism, at least combatted exterminatory settler discourses and practices at multiple sites of empire, and provided spaces on mission and protectorate stations in which indigenous peoples could be shielded to a very limited extent from dispossession
and murder. They also, unintentionally, reproduced discourses of a civilising mission and of a universal humanity that could be deployed by anticolonial nationalists in other sites of empire that were never invaded to the same extent by settlers, in independence
struggles from the mid-twentieth century. Finally, as Whatmore’s (2002) analysis of the Select Committee on Aborigines reveals, they provided juridical narratives that are part of the arsenal of weapons that indigenous peoples can wield in attempts to claim redress

The politics of humanism in practice, then, was riddled with contradiction,


and recompense in a postcolonial world.

fraught with particularity and latent with varying possibilities . It could be relatively progressive and
liberatory; it could be dispossessive and culturally genocidal . Within its repertoire lay potential to combat environmental and biological
determinism and innatism, however, and this should not be forgotten in a rush to condemn humanism’s universalism

as well as its anthropocentrism. It is in the tensions within universalism that the ongoing potential of an always

provisional, self-conscious, flexible and strategic humanism – one that now recognises the continuity
between the human and the non-human as well as the power-laden particularities of the male,
middle class, Western human subject – resides.

Calling for the “end of the world” is an obfuscatory and counter-productive


alternative. There’s no singular world, and they misread Fanon’s politics as an
abstract anti-humanism rather than a nuanced and political anti-colonialism.
The alt solidifies an anti-black “white” world as inevitable, precluding the
invention of new worlds and the transformation of existing ones.
Thomas 18 – Greg Thomas, Associate Professor of English at Tufts (“Afro-Blue Notes: The
Death of Afro-pessimism (2.0)?,” Theory & Event, Vol. 21, no. 1)

Thus there is the serious problem of elliptical truncation in Wilderson's repeated quotation of
the "end of the world" line taken from Fanon's Black Skin, White Masks. The "world" is never so
generic and singular as pessimism would have it, whether in or outside this or that Fanon—
whether it is the critical but "French" colonial Fanon or the radically decolonizing Fanon who
wages pan-African revolt against the French and all colonialism. The younger Fanon wrote, "The
Martinican is a man crucified. …[M]y friend had fulfilled in a dream his wish to become white—
that is, to be man. …I will tell him, 'The environment, society are responsible for your delusion.'
Once that has been said, the rest will follow of itself, and what that is we know. The end of the
world."39 The "world" in question is quite a specific one. It is not the only world that is, or ever
was, before another must be created into being out of necessity. It is the white world that
represents itself "as if" (to borrow a turn of phrase from Wynter here) it were the only world in
truth.

The Fanon of Black Skin, White Masks was writing about a certain type or class of "Negro" who
comes to crave this world. The introduction states this plainly from the start: "I shall try to
discover the various attitudes that the Negro adopts in contact with the white civilization"—that
"world." "The 'jungle savage' is not what I have in mind," this Fanon says with the arrogance of
the colonized elite, the assimilé or the so-called evolué: "That is because for him certain factors
have not yet acquired importance."40 He is dealing with the "climbers"41—in social class terms.
If, notably, to "speak a language is to take on a world,"42 the "accursed" world of the Antilles is
left behind by the "non-jungle savage" who strives, "climbs," in order to become truly part of
the "French" world of France in his or her captured imagination. His or her portrait is exposed in
"The Negro and Language," [End Page 298] "The Fact of Blackness" (or "The Lived Experience of
Blacks"), etc. In this case, he who can identify the need to restructure the world ends up instead
getting restructured by one world himself. He moves to this other, "spatial and temporal world"
of the white world—wanting, lusting for colonial liberal humanist acceptance, thinking "reason"
or "rationality" can be his salvation into "recognition." The whole world dominated by the white
world is the only world that this genre of "Negro" and the white world values, a pivotal
distinction effaced by Afro-pessimism (2.0). But, of course, he finds nullification in that world of
his cultivated longings. After all, a "normal Negro child, having grown up within a normal family,
will become abnormal on the slightest contact with the white world."43 Moreover, "[in] the
white world," Fanon writes, the "man of color" encounters "difficulty in the development of his
bodily schema."44 He would experience his body as a "corporal malediction" insofar as he
identifies this really white world as the only "real" world that could really, possibly matter to him
and to all real, rational "men" for whom jungles and savagery are merely things of the past.45

Négritude tempts the early Fanon's search for a "manhood" shared in the West with another,
historical take on worlds: "The white man wants the world; he wants it for himself alone. …Like
a magician, I robbed the white man of 'a certain world,' forever after lost to him and his."46 Let
this not be deleted from canonical-myopic readings of Black Skin, White Masks—qua "Fanon." It
was inspired by Léopold Senghor. But it is Césaire who provides a general resource-book for
Fanon's worldings here. The reference to Return to My Native Land is extensive in this regard.
He would tell Jean-Paul Sartre, accordingly: "There will always be a world—a white world—
between you and us."47 (92). The Césaire text shifts to "The Rebel," however, in The Wretched
of the Earth when Fanon is no longer ashamed to identify with the Haitian Revolution as he
was in Black Skin, White Masks ("I am a man. …I am not solely responsible for the revolt in Santo
Domingo"48), as a soon-to-be world-famous and world-infamous champion of Algerian and
African Revolution himself. That is when "civilization" and "savagery" will have come to mean
extremely different things for the iconic anti-colonialist Black revolutionary psychiatrist with a
hospital now named for him in Blida, a militant ambassadorship to Ghana behind him, a
neuropsychiatric day center in Tunisia cofounded by him, and a national archive now named for
him in Algiers, among other things and other places. Indeed, this is how his decolonizing praxis
would essentially rewrite his "Antillais et Africains" article from earlier years.

All told, Fanon made substantial reference to the Black world, the African world as well as the
Arab world over and against the white world, "the settler's world," the colonial world ("cut in
two"), the Western world, and much more. The white world masquerading as [End Page 299]
"the world," proper, is clearly exposed to be a certain world "outside"—the world of the
"foreigner."49 The worlds of "Third Worldism" mean to upstage it at long last. The actively
"underdeveloped world" will "shake" the world whose opulence is steeped in "slavery" and its
"blood,"50 destroying the colonizing zoning of Europeanism not by "rational confrontation" or
conversation but through "counter-violence."51 The move charted by the ultimate Fanon (and
the bulk of Fanon) is no longer of the "climber" from "jungle" to "mother country" "manhood"
but from colonized "individual" to liberated "nation" on to the un-whitened "world" at large
that must be expressly expropriated from European appropriation:52 "It is a question of the
Third World starting a new history or Man,"53 Fanon writes. The Wretched of the Earth cannot
be quarantined from Fanon's discourse on human being and "humanism;" it is the climax of that
discourse on colonial imperialism as well as slavery, for such colonialism and slavery form an
inseparable complex of physical and metaphysical (or "psycho-affective") assaults on humanity
beyond the white West. Hence, it is not the end of some worlds that Fanon desires or
pronounces as he speaks in their name to end the specific world held onto tightly in critique by
pessimists. Routinely, white discourse becomes the only discourse there is to discuss in their
own rhetorical articulations. To shear the white world of this spatial and temporal , cultural and
historical specificity reinforces whiteness and "anti-Blackness," so to speak; it reinforces
"Eurocentrism" or Europeanism and naturalizes by systematically totalizing the "anti-Black"
power of a provincial and solipsistic white-supremacism; and, in this reinforcement, the
elliptical truncation of the "end of the world" discourse in Fanon's Black Skin, White Masks and
more radically so elsewhere provides an explanation for what Sexton cannot fathom in "The
Social Life of Social Death" (2011), that is, how or why "Afro-pessimism" (2.0) could be so curtly
dismissed by many critics as another "anti-Black" discourse itself.
1AR
1AR Solvency
Omolade is an unequivocal aff author—advocates for political agitation and
challenging US militarism
Omolade 84—Barbara Omolade, PhD in Sociology from The City University of New York, chair
of the Civil Rights Coordinating Council (“Women of Color and the Nuclear Holocaust,” Women’s
Studies Quarterly, Vol. 12, No. 2, Teaching about Peace, War, and Women in the Military
(Summer, 1984), p. 12)

Nuclear arsenals and nuclear power are part of a rational and holistic system in which those in power
hold power over all aspects of world society. It is irrelevant whether they are called "mad" or "sane" by protestors
and critics of the system. The fact remains that they are men, initiating and carrying out the dictates of a rational system of military
terror. Calling them "mad," or considering them military "male chauvinists," assures only that the rational system they are part of
will remain obscure, and that the responsibility of each man in the Pentagon will never be understood clearly enough to wage an
effective political struggle against it. Nuclear
disarmament and peace are political questions requiring
political solutions of accountability and struggle around who has the power to determine the
destiny of the earth. The demand for unconditional U.S. disarmament holds that the U.S.
government is responsible for its actions and should be held accountable for them.

Das könnte Ihnen auch gefallen